INSIGHTSIAS SIMPLYFYING IAS EXAM PREPARATION

OFFLINE Centres at BENGALURU | DELHI | HYDERABAD

INSTA Revision Plan 3.0 - 2020

INSTA Tests

DAYS 13 to 16

SOLUTIONS

For more visit: www.INSIGHTSONINDIA.com Copyright © by Insights IAS All rights are reserved. No part of this document may be reproduced, stored in a retrieval system or transmitted in any form or by any means, electronic, mechanical, photocopying, recording or otherwise, without prior permission of Insights IAS.

INSIGHTSIAS SIMPLYFYING IAS EXAM PREPARATION

DAY – 13

1. Arrange the following hills from East to West 1. hills 2. Ramgarh hills 3. Ajanta hills 4. Select the correct answer using the code given below: (a) 2 4 1 3 (b) 2 3 1 4 (c) 1 2 4 3 (d) 1 2 3 4

Solution: C

2. Which of the following passes connects with ? 1. 2. Lipu Lekh 3. Pass Select the correct answer using the code given below: (a) 1 and 2 only www.insightsonindia.com 1 INSTA Revision 3.0 INSIGHTSIAS SIMPLYFYING IAS EXAM PREPARATION

(b) 1 and 3 only (c) 2 and 3 only (d) 1, 2 and 3

Solution: D

• Nathu La Pass It is located in the state of . This famous pass is located in the - China border was reopened in 2006. It forms a part of an offshoot of the ancient silk route. It is one of the trading border posts between India and China.

• Shipki La Pass It is located through Gorge. It connects with . It is India’s third border post for trade with China after Lipu Lekh and Nathula Pass.

Pass This pass passes through the . It connects Sikkim with , the capital of Tibet.

• Chang-La It is a high in the Greater . It connects with Tibet.

: and Parvati Valley It joins Spiti Valley and Parvati Valley. It is a high mountain pass in between the and Spiti of Himachal Pradesh. It is a bypass route of Pin-Parvati Pass.

: Kullu-Lahul-Spiti This is located in the state of Himachal Pradesh. It has excellent road transportation. This pass connects Kullu, Spiti, and Lahul.

• Bomdi-La: Arunachal Pradesh-Lhasa The Bomdi-La pass connects Arunachal Pradesh with Lhasa, the capital city of Tibet. It is located in the east of .

: Srinagar- Kargil & It connects Srinagar with Kargil and Leh. Beacon Force of Border Road Organization is responsible for clearing and maintaining the road, especially during the winter.

• Lipu Lekh: -Tibet It is located in Uttarakhand. It connects Uttarakhand with Tibet. This pass is an important border post for trade with China. The for Manasarovar travel through this pass.

3. Consider the following statements 1. A substantial portion of India’s requirement of edible oil is met through import of palm oil. 2. India’s vegetable oil economy is world’s fourth largest after USA, China & Brazil. 3. Palm oil contributes 30% of vegetable oil import of India www.insightsonindia.com 2 INSTA Revision 3.0 INSIGHTSIAS SIMPLYFYING IAS EXAM PREPARATION

Which of the statements given above is/are correct? (a) 1 and 3 only (b) 3 only (c) 2 and 3 only (d) 1 and 2 only

Solution: D

India is one of the major oilseeds grower and importer of edible oils. India’s vegetable oil economy is world’s fourth largest after USA, China & Brazil. A substantial portion of India’s requirement of edible oil is met through import of palm oil. Palm oil contributes 70% of vegetable oil import and is one of the cheapest oil due to high productivity per hectare. India is the world’s largest importer of palm oil, driving 23 per cent of total global demand from plantations in Indonesia and Malaysia. https://www.thehindu.com/business/avoid-oil-imports-from-malaysia-trade- body/article29760672.ece

4. Consider the following statements 1. The Thal is located on route. 2. Pal Ghat joins the Madurai city in Tamil Nadu with Kottayam district in Kerala. Which of the statements given above is/are correct? (a) 1 only (b) 2 only (c) Both 1 and 2 (d) Neither 1 nor 2

Solution: A

Shencottah Gap: Madurai-Kottayam It is located in the Western . It joins the Madurai city in Tamil Nadu with Kottayam district in Kerala.

• The second largest gap in which is situated five kilometres from town is known by its name that is Shencottah Gap road rail lines pass through this gap which connect Shencottah with Punalur.

www.insightsonindia.com 3 INSTA Revision 3.0 INSIGHTSIAS SIMPLYFYING IAS EXAM PREPARATION

Bhor Ghat or Bor Ghat or Bhore Ghaut is a mountain passage located between Palasdari and for railway and between Khopoli and Khandala on the road route in , India situated on the crest of the western Ghats. It is located at an elevation of four hundred and forty-one meters’ elevation above sea level.

• The ghat has a bit of historical evidence. The ghat was the ancient route developed by Satavahana to connect the ports of Choul, Revdanda Panvel etc. on the coast and the surrounding areas on the Deccan plateau. Today the ghat plays a massive part of the Great Indian Peninsula Railway laid from Mumbai to Pune. Thal Ghat (also called Thul Ghat or Ghat) is a ghat section (mountain incline or slope) in the Western Ghats near the town of Kasara in Maharashtra. The Thal Ghat is located on the busy Mumbai–Nashik route, and is one of the four major routes, rail and road routes, and leading into Mumbai. The railway line, which passes through the ghat is the steepest in India with a gradient of 1 in 37 Pal Ghat

• The is located in the Western Ghats between the states of Tamil Nadu and Kerala. India at an elevation of about 140 m. The mountain pass is located between Nilgiri Hills in the north and towards the south and connects Coimbatore in Tamil Nadu with Palakkad in Kerala. The mountain pass was an important instrument for human migration across India’s southern tip throughout settled history.

5. Which of the following parameters is/are considered to rank institutions across India under National Institute ranking framework (NIRF)? 1. Teaching learning and resources 2. Research, consulting and collaborative performance 3. Outreach and inclusivity Select the correct answer using the code given below (a) 2 only (b) 1 and 3 only (c) 3 only (d) 1, 2 and 3

Solution: D

National Institute Ranking Framework (NIRF) is a methodology to rank institutions across the country.

www.insightsonindia.com 4 INSTA Revision 3.0 INSIGHTSIAS SIMPLYFYING IAS EXAM PREPARATION

It provides credible official rankings of India’s higher educational institution on a set of parameters. The National Institutional Ranking Framework (NIRF) was approved by the MHRD and launched by the Minister of Human Resource Development on September 29, 2015. The parameters are under the five broad headings: 1. Teaching learning and resources 2. Research, consulting and collaborative performance 3. Graduation outcomes 4. Outreach and inclusivity 5. Perception

6. Which of the following is/are artificial lakes? 1. Kodaikanal Lake 2. Bhojtal Lake 3. Kankaria Lake 4. Hamirsar Lake Select the correct answer using the code given below: (a) 1 and 2 only (b) 3 and 4 only (c) 1, 2 and 4 only (d) 1, 2, 3 and 4

Solution: D

Hamirsar Lake

• Type: Artificial Lake

• Surface area: 11 Hectare

• State: Gujarat

Kankaria Lake

• Surface area: Artificial Lake

• State: Gujarat

www.insightsonindia.com 5 INSTA Revision 3.0 INSIGHTSIAS SIMPLYFYING IAS EXAM PREPARATION

Artificial Lakes are constructed for industrial or agricultural use, for hydro-electric power generation or domestic water supply, or for aesthetic or recreational purposes.

• Guru Gobind Sagar is an example of an artificial lake. It supports the Bhakra Nangal Hydel Project.

• Kodaikanal Lake in Tamil Nadu is another example of Artificial (manmade) Lake.

• Bhojtal Lake- Madhya Pradesh › Also known as Upper Lake lies on the western side of the capital city of Madhya Pradesh, Bhopal. › The largest artificial lake in Asia.

7. Consider the following statements: 1. Lake Superior is the world’s largest freshwater lake in terms of volume. 2. Lake Baikal is the world’s deepest lake. 3. The Dead Sea is the saltiest body of water on Earth. Which of the statements given above is/are correct? (a) 2 only (b) 1 and 3 only (c) 2 and 3 only (d) 1, 2 and 3

Solution: A

Lake Baikal is the world’s largest freshwater lake in terms of volume. Lake Baikal is the largest freshwater lake in the world (by volume) and the world’s deepest lake. The salty Caspian Sea has the greatest surface area of any lake at 143,200 square miles (370,886 square kilometers). The Dead Sea, nestled on the borders of Jordan and Israel. The water here is around 10 times saltier than sea water. However, it is only the fifth saltiest body of water on Earth.

8. Consider the following statements regarding Central Electricity Regulatory Commission (CERC) 1. It is an autonomous body under Ministry of Power www.insightsonindia.com 6 INSTA Revision 3.0 INSIGHTSIAS SIMPLYFYING IAS EXAM PREPARATION

2. It was instituted primarily to regulate the tariff of Power Generating companies. Which of the statements given above is/are correct? (a) 1 only (b) 2 only (c) Both 1 and 2 (d) Neither 1 nor 2

Solution: B

• CERC is a statutory body functioning under section 76 of the Electricity Act 2003. It functions with a quasi-judicial status under the Ministry of Power.

• CERC was initially constituted under the Electricity Regulatory Commissions Act, 1998 on 24th July, 1998. Functions

• CERC was instituted primarily to regulate the tariff of Power Generating companies (government owned as well as other companies with interstate transmission.) It works for o Rationalization of electricity tariffs o Transparent policies regarding subsidies o Promotion of efficient and environmentally benign policies o Electricity Tariff regulation

9. Arrange the following cities from west to east 1. Nagpur 2. Ujjain 3. Udaipur 4. Junagadh Select the correct answer using the code given below: (a) 4 1 3 2 (b) 4 3 2 1 (c) 1 4 2 3 (d) 1 3 2 4 www.insightsonindia.com 7 INSTA Revision 3.0 INSIGHTSIAS SIMPLYFYING IAS EXAM PREPARATION

Solution: B

10. Consider the following statements regarding Traditional Knowledge Digital Library (TKDL) 1. It is a community based informal traditional knowledge repository 2. It is supported by Ministry of AYUSH 3. TKDL acts as a bridge between the traditional knowledge information existing in local languages and the patent examiners at IPOs. Which of the statements given above is/are correct? (a) 1 and 2 only (b) 3 only (c) 1 and 3 only (d) 2 and 3 only

Solution: D

Traditional Knowledge Digital Library (TKDL) is an Indian digital knowledge repository of the traditional knowledge, especially about medicinal plants and formulations used in Indian systems of medicine.

• It is a pioneer initiative of India to prevent misappropriation of country’s traditional medicinal knowledge at International Patent Offices. It is a collaborative project of the Council of Scientific and Industrial Research (CSIR) and Ministry of AYUSH. www.insightsonindia.com 8 INSTA Revision 3.0 INSIGHTSIAS SIMPLYFYING IAS EXAM PREPARATION

• TKDL acts as a bridge between the traditional knowledge information existing in local languages and the patent examiners at IPOs.

• The focus of TKDL was on breaking the language and format barriers by scientifically converting and structuring the available TK in International Patent Classification.

• The TKDL contains documentation of publicly available traditional knowledge (TK) that relates to Ayurveda, Unani, Siddha and Yoga and is in digitized format.

11. Arrange the following cities from North to South 1. Shimla 2. Dehradun 3. Agra 4. New Delhi Select the correct answer using the code given below: (a) 2 4 1 3 (b) 2 3 1 4 (c) 1 2 4 3 (d) 1 2 3 4

Solution: C

www.insightsonindia.com 9 INSTA Revision 3.0 INSIGHTSIAS SIMPLYFYING IAS EXAM PREPARATION

12. Consider the following islands of Andaman and Nicobar islands: 1. Barren Island 2. Little Andaman 3. Car Nicobar Island Arrange the above given islands in the direction from South to North: (a) 3-2-1 (b) 3-1-2 (c) 1-2-3 (d) 1-3-2

Solution: A

www.insightsonindia.com 10 INSTA Revision 3.0 INSIGHTSIAS SIMPLYFYING IAS EXAM PREPARATION

13. Consider the following statements regarding Safe City Project 1. It is a comprehensive plan with the purpose of strengthening safety and security of women in public places 2. It will be implemented in few selected cities under Nirbhaya Fund. Which of the statements given above is/are correct? (a) 1 only (b) 2 only (c) Both 1 and 2 (d) Neither 1 nor 2

Solution: C

Safe City Project is a comprehensive plan with the purpose of strengthening safety and security of women in public places. It will be implemented in 8 selected cities under Nirbhaya Fund. The eight cities include Mumbai, Delhi, , Chennai, Bengaluru, Hyderabad, Ahmedabad and Lucknow. The project would be implemented as a Centrally Sponsored Scheme with Centre and State sharing the funding in 60:40 ratio.

14. Consider the following statements regarding the Andhra Pradesh state: 1. The state has the longest coastline among all the states of India. 2. Rice is a major food crop in Andhra Pradesh, contributing about 77 percent of the food grain production. Which of the statements given above is/are correct? (a) 1 only (b) 2 only (c) Both 1 and 2 (d) Neither 1 nor 2

Solution: B

Andhra Pradesh

• The state has the second longest coastline of 974 km (605 mi) among all the states of India, second only to Gujarat. www.insightsonindia.com 11 INSTA Revision 3.0 INSIGHTSIAS SIMPLYFYING IAS EXAM PREPARATION

• Agriculture is the main occupation of about 62 per cent of the people in the state. Rice is a major food crop and staple food contributing about 77 percent of the food grain production.

• Andhra Pradesh is situated on the country’s southeastern coast. It is the eighth largest state in the country covering an area of 160,205 sq. km. According to 2011 census, the state is tenth largest by population, with 49,386,799 inhabitants.

15. Global Economic Prospects is released by (a) United Nations (b) International Monetary Fund (c) World Economic Forum (d) None of the above

Solution: D

The Global Economic Prospects (GEP) is the World Bank’s semi-annual flagship publication on the state of the world economy. In a nutshell, the outlook for the global economy has darkened, amid slowing activity and heightened downside risks.

16. Consider the following statements regarding the Himachal Pradesh State: 1. It is called the “Fruit Bowl” of the country. 2. The territory of this state has parts of the Shivaliks, Inner Himalayas and the Greater Himalayas. 3. Tourism contributes about 25 per cent to state GDP. Which of the statements given above is/are correct? (a) 1 and 2 only (b) 1 and 3 only (c) 2 and 3 only (d) 1, 2 and 3

Solution: A

Himachal Pradesh

• Himachal Pradesh, a beautiful land is situated in the lap of the western Himalayas, located between 30°.22’ and 33°. 12’ North latitude and between www.insightsonindia.com 12 INSTA Revision 3.0 INSIGHTSIAS SIMPLYFYING IAS EXAM PREPARATION

75°.47’ and 79°. 41 East longitude. This wonderful land inhabited by people of various castes and religion groups, is also called as “Dev Bhumi: the abode of gods and goddesses”. Himachal drives its name from the Himalayas, literally means “land of snowy mountains”. The territory of this state can be divided into three zones, Outer Himalayas or the Shivaliks, Inner Himalayas or Mid- Mountains and the Greater Himalayas or Alpine Zone. Agriculture

• Agriculture provides direct employment to about 70 per cent of the main working population of the state. Income from agriculture and allied sectors accounts for nearly 22.1 per cent of the total state domestic product. Out of the total geographical area of 55, 673 sq. km., area of operational holding is about 9.79 lakh hectares. Horticulture

• Himachal Pradesh is called the “Fruit Bowl” of the country. Horticulture plays an important role in the agrarian economy. Forestry

• Forest in Himachal Pradesh covers an area of 37, 033 sq. km which comes to 66.52 per cent of the total geographical area of the state. The strategy of government in forestry management is conservation along with rational utilization and side by side expanding its base. Forest plantation is being carried out under productive forestry scheme and soil conservation schemes. Tourism contributes 6.8 per cent to state GDP and it is set to increase.

17. Consider the following statements regarding the Chumbi valley: 1. It is located in the state of Uttarakhand. 2. It is formed by the passage of the River Bhagirathi. Which of the statements given above is/are correct? (a) 1 only (b) 2 only (c) Both 1 and 2 (d) Neither 1 nor 2

Solution: D

Chumbi Valley, valley in the eastern Great Himalaya Range of the southern , China. It is situated on a small south-pointing protuberance of territory between Bhutan (east) and Sikkim state, India (west). Formed by the passage of the Amo (Torsa) River, which rises below Tang Pass and flows south into www.insightsonindia.com 13 INSTA Revision 3.0 INSIGHTSIAS SIMPLYFYING IAS EXAM PREPARATION

Bhutan, the valley has an average elevation of 9,500 feet (2,900 metres), forested slopes, and a pleasant climate most of the year. It is connected to Sikkim to the southwest through the mountain passes of Nathu La and Jelep La. It is a disputed territory between China and Bhutan. Important for prelims because of standoff.

18. Consider the following statements regarding Bank Boards Bureau (BBB) 1. It appoints Board of Directors in PSBs and Financial Institutions. 2. It builds a data bank containing data relating to the performance of PSBs/FIs, its senior management. Which of the statements given above is/are correct? (a) 1 only (b) 2 only (c) Both 1 and 2 (d) Neither 1 nor 2

Solution: C

• Banks Board Bureau is an autonomous body of the tasked to improve the governance of Public Sector Banks.

• It was set up under the government’s Indradhanush programme. It was conceived by the PJ Nayak committee. Composition

• BBB has representatives from government and RBI apart from independent banking professionals. It is housed in RBI’s Central Office in Mumbai. Functions of the Banks Board Bureau:

• Selection and appointment of Board of Directors in PSBs and Financial Institutions (Whole-time Directors and Non-Executive Chairman);

• Advising the central government in matters of banking governance at the higher levels and their training and management.

• Building a data bank containing data relating to the performance of PSBs/FIs, its senior management and the Board of Directors and share the same with Government;

19. Rutland Island, recently seen in news, is located in (a) Andaman and Nicobar www.insightsonindia.com 14 INSTA Revision 3.0 INSIGHTSIAS SIMPLYFYING IAS EXAM PREPARATION

(b) Lakshadweep (c) Gulf of Kutch (d) Ganga Brahmaputra Delta

Solution: A

Rutland Island is an island of the Andaman Islands. It belongs to the South Andaman administrative district. The National Board of Wildlife (NBWL) has approved Rutland Island in South Andaman to be the site for the country’s long-range missile test facility.

20. With reference to the Renewable Energy Certificates (REC), consider the following statements 1. It is a mechanism is a market based instrument to promote renewable sources of energy and development of market in electricity. 2. RECs are not eligible to trade in India 3. RECs represent 1mw-hour of power produced from a renewable energy source Which of the statements given above is/are correct? (a) 2 only (b) 1 and 3 only (c) 2 and 3 only (d) 1 only

Solution: B

The Renewable Energy Certificates (REC) mechanism is a market based instrument to promote renewable sources of energy and development of market in electricity. Utilities that are unable to fulfil their RPO can meet their obligations by buying certificates known as Renewable Energy Certificates (REC). RECs represent 1mw- hour of power produced from a renewable energy source and are tradable at power exchanges. It is classified into Solar REC and non-solar REC.REC market was launched in 2010. National Load Despatch Centre (NLDC) has been designated as the Central Agency for coordination and implementation of Renewable Energy Certificate Mechanism. www.insightsonindia.com 15 INSTA Revision 3.0 INSIGHTSIAS SIMPLYFYING IAS EXAM PREPARATION

In India, RECs are traded on the Indian Energy Exchange (IEX) and the Power Exchange of India Ltd (PXIL).

21. Consider the following pairs of places in news and their states: Place State 1. Belum caves : Madhya Pradesh 2. Lothal : Gujarat 3. Bogibeel Bridge : Arunachal Pradesh Which of the pairs given above is/are correctly matched? (a) 1 and 3 only (b) 2 only (c) 2 and 3 only (d) 1 and 2 only

Solution: B

The Belum Caves is the largest and longest cave system open to the public on the Indian subcontinent, known for its speleothems, such as stalactite and stalagmite formations. The Belum Caves have long passages, galleries, spacious caverns with fresh water and siphons. It is located in the state of Andhra Pradesh. https://www.thehindu.com/news/national/andhra-pradesh/belum-caves-festival- postponed/article30413624.ece

Lothal was one of the southernmost cities of the ancient Indus Valley Civilization, located in the Bhal region of the modern state of Gujarat. https://www.thehindubusinessline.com/news/science/indias-first-maritime- museum-coming-up-at-lothal-in-gujarat/article30139189.ece

Bogibeel bridge is a combined road and rail bridge over the Brahmaputra River in the northeastern Indian state of Assam https://www.livemint.com/Politics/zMhowFzkONQkLllh9ycZtK/PM-Modi- inaugurates-Bogibeel--longest-railroad-brid.html

22. Koundinya Wildlife Sanctuary, sometimes seen in news, is located in: (a) Karnataka (b) Kerala (c) Tamil Nadu www.insightsonindia.com 16 INSTA Revision 3.0 INSIGHTSIAS SIMPLYFYING IAS EXAM PREPARATION

(d) Andhra Pradesh

Solution: D

Kaundinya Wildlife Sanctuary is a wildlife sanctuary and an elephant reserve situated in Andhra Pradesh, India. It is the only sanctuary in Andhra Pradesh with a population of Asian elephants, which migrated after 200 years from neighbouring regions. https://www.thehindu.com/news/national/andhra-pradesh/4-poachers-nabbed- in-koundinya-wildlife-sanctuary/article32096013.ece

23. Consider the following statements regarding Tiger Census, 2018 1. Tiger census conducted for every four years 2. Both cubs and adult tigers are counted in the census 3. It is conducted under the overall guidance of Global Tiger Forum Which of the statements given above is/are correct? (a) 1 only (b) 1 and 2 only (c) 1, 2 and 3 (d) 1 and 3 only

Solution: A

Tiger Census 2018

• It is a four-yearly report, which gives out the number of big cats living in the country.

• This is the fourth cycle of the tiger census. The first was conducted in 2006, second in 2010 and third in 2014.

• In the census, only adult tigers are counted and do not include cubs. This is the first time neighboring , Bangladesh and Bhutan are being roped in for the census because they constitute a larger tiger range in the Indian subcontinent.

• It is conducted under the overall guidance of NTCA and Ministry of Environment, Forest and Climate Change.

www.insightsonindia.com 17 INSTA Revision 3.0 INSIGHTSIAS SIMPLYFYING IAS EXAM PREPARATION

24. Consider the following statements regarding the Andaman and Nicobar Islands: 1. Little Andaman is separated from the Nicobar Islands by the Ten Degree Channel. 2. Saddle Peak is located on Car Nicobar Island. 3. Narcondam volcanic island is located south of Barren Island. Which of the statements given above is/are correct? (a) 1 only (b) 2 only (c) 1 and 3 only (d) 1, 2 and 3

Solution: A

www.insightsonindia.com 18 INSTA Revision 3.0 INSIGHTSIAS SIMPLYFYING IAS EXAM PREPARATION

Ten degrees Channel which lies between Little Andaman Island and the Car Nicobar. It is so named as it lies on the 10-degree line of latitude, north of the equator. Saddle Peak or Saddle Hill is located on North Andaman Island in India’s Andaman and Nicobar Islands. At 731 m, it is the highest point of the archipelago in the Bay of Bengal. It is surrounded by Saddle Peak National Park

25. Dhimsa, a tribal dance, is largely practiced in which of the following region of India? (a) Northern India (b) Eastern India (c) Western India (d) North-Eastern India

Solution: B

Dhimsa is a tribal dance performed by the Porja’ caste women. The dance is originated in Odisha but has almost become a dance of Visakhapatnam, Andhra Pradesh. It is performed by a group of 15–20 women in a circle – praising the deity for the welfare of their domestic life or at weddings for a happy married life.

DAY – 14

26. Consider the following statements regarding Effects of El Nino 1. Fish catches off the coast of South America were lower than in the normal year. 2. Heavy rains in California, Ecuador, and the Gulf of Mexico. 3. Severe droughts occur in Australia, Indonesia, India and southern Africa. Which of the statements given above is/are correct? (a) 1 and 2 only (b) 1 and 3 only (c) 2 and 3 only (d) 1, 2 and 3

Solution: D www.insightsonindia.com 19 INSTA Revision 3.0 INSIGHTSIAS SIMPLYFYING IAS EXAM PREPARATION

Effects of El Nino

• The warmer waters had a devastating effect on marine life existing off the coast of Peru and Ecuador.

• Fish catches off the coast of South America were lower than in the normal year (Because there is no upwelling).

• Severe droughts occur in Australia, Indonesia, India and southern Africa.

• Heavy rains in California, Ecuador, and the Gulf of Mexico. How El Nino impacts monsoon rainfall in India

• El Nino and Indian monsoon are inversely related.

• The most prominent droughts in India – six of them – since 1871 have been El Nino droughts, including the recent ones in 2002 and 2009

• However, not all El Nino years led to a drought in India. For instance, 1997/98 was a strong El Nino year but there was no drought (Because of IOD).

• On the other hand, a moderate El Nino in 2002 resulted in one of the worst droughts.

• El Nino directly impacts India’s agrarian economy as it tends to lower the production of summer crops such as rice, sugarcane, cotton and oilseeds.

• The ultimate impact is seen in the form of high inflation and low gross domestic product growth as agriculture contributes around 14 per cent to the Indian economy.

27. Consider the following statements regarding Indian Ocean Dipole 1. The Indian Ocean Dipole (IOD) is defined by the difference in sea surface temperature between two areas – a western pole in the Arabian Sea and an eastern pole in the eastern Indian Ocean. 2. Negative IOD results in stronger than usual cyclonogenesis in Arabian sea. Which of the statements given above is/are correct? (a) 1 only (b) 2 only (c) Both 1 and 2 (d) Neither 1 nor 2

Solution: A

www.insightsonindia.com 20 INSTA Revision 3.0 INSIGHTSIAS SIMPLYFYING IAS EXAM PREPARATION

Indian Ocean Dipole effect

• However, it was discovered that just like ENSO was an event in the Pacific Ocean, a similar seesaw ocean-atmosphere system in the Indian Ocean was also at play. It was discovered in 1999 and named the Indian Ocean Dipole (IOD).

• The Indian Ocean Dipole (IOD) is defined by the difference in sea surface temperature between two areas (or poles, hence a dipole) – a western pole in the Arabian Sea (western Indian Ocean) and an eastern pole in the eastern Indian Ocean south of Indonesia.

• IOD develops in the equatorial region of Indian Ocean from April to May peaking in October.

• With a positive IOD wind over the Indian Ocean blow from east to west (from Bay of Bengal towards Arabian Sea). This results in the Arabian Sea (western Indian Ocean near African Coast) being much warmer and eastern Indian Ocean around Indonesia becoming colder and dry.

• In the negative dipole year (negative IOD), reverse happens making Indonesia much warmer and rainier.

• It was demonstrated that a positive IOD index often negated the effect of ENSO, resulting in increased Monsoon rains in several ENSO years like the 1983, 1994 and 1997.

• Further, it was shown that the two poles of the IOD – the eastern pole (around Indonesia) and the western pole (off the African coast) were independently and cumulatively affecting the quantity of rains for the Monsoon in the Indian subcontinent.

• Similar to ENSO, the atmospheric component of the IOD was later discovered and named as Equatorial Indian Ocean Oscillation [EQUINOO][Oscillation of warm water and atmospheric pressure between Bay of Bengal and Arabian Sea]. Impact on IOD on Cyclonogeneis in Northern Indian Ocean

• Positive IOD (Arabian Sea warmer than Bay of Bengal) results in more cyclones than usual in Arabian Sea.

• Negative IOD results in stronger than usual cyclonogenesis (Formation of Tropical Cyclones) in Bay of Bengal. Cyclonogenesis in Arabian Sea is suppressed.

28. Gale crater, sometimes seen in news, is situated on the planet: (a) Mercury (b) Venus (c) Jupiter (d) Mars

www.insightsonindia.com 21 INSTA Revision 3.0 INSIGHTSIAS SIMPLYFYING IAS EXAM PREPARATION

Solution: D

Gale is a crater, and probable dry lake, on Mars. It is 154 km in diameter and estimated to be about 3.5-3.8 billion years old. Scientists chose Gale Crater as the landing site for Curiosity because it has many signs that water was present over its history. https://www.firstpost.com/tech/science/scientists-think-they-know-what-the- source-of-methane-in-mars-atmosphere-could-be-6373891.html

29. Consider the following statements regarding El Niño and El Niño Modoki 1. The El Niño Modoki phenomenon is characterized by the anomalously warm central equatorial Pacific flanked by anomalously cool regions in both west and east. 2. Conventional El Niño is characterized by strong anomalous cooling in the eastern equatorial Pacific. Which of the statements given above is/are correct? (a) 1 only (b) 2 only (c) Both 1 and 2 (d) Neither 1 nor 2

Solution: A

The El Niño Modoki

• El Niño Modoki is a coupled ocean-atmosphere phenomenon in the tropical Pacific.

• It is different from another coupled phenomenon in the tropical Pacific namely, El Niño.

• Conventional El Niño is characterized by strong anomalous warming in the eastern equatorial Pacific.

• Whereas, El Niño Modoki is associated with strong anomalous warming in the central tropical Pacific and cooling in the eastern and western tropical Pacific. El Niño Modoki Impacts

• The El Niño Modoki phenomenon is characterized by the anomalously warm central equatorial Pacific flanked by anomalously cool regions in both west and east. www.insightsonindia.com 22 INSTA Revision 3.0 INSIGHTSIAS SIMPLYFYING IAS EXAM PREPARATION

• Such zonal gradients result in anomalous two-cell Walker Circulation over the tropical Pacific, with a wet region in the central Pacific.

30. Consider the following statements regarding the Environment Pollution Control Authority (EPCA): 1. It is mandated by the High Court of Delhi tasked with taking various measures to tackle air pollution in the National Capital Region. 2. It was notified in 1998 by Environment Ministry under the Air (Prevention and Control of Pollution) Act, 1981. Which of the statements given above is/are correct? (a) 1 only (b) 2 only (c) Both 1 and 2 (d) Neither 1 nor 2

Solution: D

EPCA comes out with parking management plan for Delhi. In report, agency flags free parking on public land, multiplicity of agencies to be key cause of congestion and parking menace. About Environment Pollution Control Authority (EPCA):

• EPCA was constituted with the objective of ‘protecting and improving’ the quality of the environment and ‘controlling environmental pollution’ in the National Capital Region. The EPCA also assists the apex court in various environment- related matters in the region.

• EPCA is Supreme Court mandated body tasked with taking various measures to tackle air pollution in the National Capital Region. It was notified in 1998 by Environment Ministry under Environment Protection Act, 1986. Composition:

• Besides the chairman, the EPCA has 14 members, some of whom are the environment secretary of the National Capital Territory of Delhi (NCT), chairperson of the New Delhi Municipal Council, transport commissioner of the NCT, the commissioners of various municipal corporations of Delhi and professors at IIT Delhi and Jawaharlal Nehru University. Functions:

• To protect and improve quality of environment and prevent and control environmental pollution in National Capital Region.

• To enforce Graded Response Action Plan (GRAP) in NCR as per the pollution levels. www.insightsonindia.com 23 INSTA Revision 3.0 INSIGHTSIAS SIMPLYFYING IAS EXAM PREPARATION https://www.insightsonindia.com/2019/04/01/environment-pollution-prevention- and-control-authority-epca/

31. Consider the following statements regarding Trade Winds 1. The trade winds are those blowing from the equatorial low-pressure belt towards the sub-tropical high-pressure areas. 2. These are confined to a region between 30°N and 30°S throughout the earth’s surface. 3. They flow as the north-eastern trades in the northern hemisphere. Which of the statements given above is/are correct? (a) 1 and 2 only (b) 1 and 3 only (c) 2 and 3 only (d) 1, 2 and 3

Solution: C

Trade Winds

• The trade winds are those blowing from the sub-tropical high-pressure areas towards the equatorial low-pressure belt.

• Therefore, these are confined to a region between 30°N and 30°S throughout the earth’s surface.

• They flow as the north-eastern trades in the northern hemisphere and the south-eastern trades in the southern hemisphere.

• This deflection in their ideally expected north-south direction is explained on the basis of Coriolis force and Farrel’s law.

• Trade winds are descending and stable in areas of their origin (sub-tropical high pressure belt), and as they reach the equator, they become humid and warmer after picking up moisture on their way.

• The trade winds from two hemispheres meet at the equator, and due to convergence they rise and cause heavy rainfall.

• The eastern parts of the trade winds associated with the cool ocean currents are drier and more stable than the western parts of the ocean.

www.insightsonindia.com 24 INSTA Revision 3.0 INSIGHTSIAS SIMPLYFYING IAS EXAM PREPARATION

32. Consider the following statements regarding Rossby Waves 1. Rossby waves are natural phenomenon in the atmosphere and oceans due to rotation of earth. 2. Rossby waves are formed when polar air moves toward the Equator while tropical air is moving poleward. Which of the statements given above is/are correct? (a) 1 only (b) 2 only (c) Both 1 and 2 (d) Neither 1 nor 2

Solution: C

Rossby Waves

• The meandering jet streams are called Rossby Waves.

• Rossby waves are natural phenomenon in the atmosphere and oceans due to rotation of earth.

• In planetary atmospheres, they are due to the variation in the Coriolis effect (When temperature contrast is low, speed of jet stream is low, and Coriolis force is weak leading to meandering) with latitude.

• Rossby waves are formed when polar air moves toward the Equator while tropical air is moving poleward.

• The existence of these waves explains the low-pressure cells (cyclones) and high-pressure cells (anticyclones).

33. Global Energy and CO2 Status Report, has been released by: (a) United Nation Environment Program (UNEP) (b) The United Nations Framework Convention on Climate Change (UNFCC) (c) The International Energy Agency (IEA) (d) International Association of Oil & Gas Producers

Solution: C

The International Energy Agency ‘s (IEA) Global Energy & CO2 Status Report provides a snapshot of recent global trends and developments across fuels, renewable sources, and energy efficiency and carbon emissions www.insightsonindia.com 25 INSTA Revision 3.0 INSIGHTSIAS SIMPLYFYING IAS EXAM PREPARATION http://www.iea.org/events/global-energy-co2-status-report

34. Consider the following statements regarding British Type Climate 1. The seasons are very distinct. 2. Rainfall occurs throughout the year with summer maxima. 3. The cool temperate western margins are under the influence of the Westerlies all-round the year. Which of the statements given above is/are correct? (a) 1 and 2 only (b) 1 and 3 only (c) 2 and 3 only (d) 1, 2 and 3

Solution: B

British Type Climate

• Westerlies come all the year round.

• There is a tendency towards an autumn or winter maximum of rainfall.

• Light snow falls in winter.

• Ports are never frozen but frosts do occur on cold nights.

• The seasons are very distinct.

• And the climate is very favorable for maximum human output. British Type Climate or Cool Temperate Western Margin Climate or North-West European Maritime Climate.

• The cool temperate western margins are under the influence of the Westerlies all-round the year.

• They are the regions of frontal cyclonic activity [Temperate Cyclones].

• This type of climate is typical to Britain, hence the name ‘British Type’.

• Also called as North-West European Maritime Climate due to greater oceanic influence. British Type Climate

• Moderately warm summers and fairly mild winters.

• Rainfall occurs throughout the year with winter maxima. www.insightsonindia.com 26 INSTA Revision 3.0 INSIGHTSIAS SIMPLYFYING IAS EXAM PREPARATION

35. Consider the following statements regarding the Outer Space Treaty, 1967: 1. States shall be responsible for national space activities carried out by governmental entities only. 2. It prohibits all kinds of weapons to be placed in outer space. 3. States shall be liable for damage caused by their space objects Which of the statements given above is/are correct? (a) 1 and 2 only (b) 3 only (c) 2 and 3 only (d) 1 only

Solution: B

The Outer Space Treaty was considered by the Legal Subcommittee in 1966 and agreement was reached in the General Assembly in the same year ( resolution 2222 (XXI)). The Treaty was largely based on the Declaration of Legal Principles Governing the Activities of States in the Exploration and Use of Outer Space, which had been adopted by the General Assembly in its resolution 1962 (XVIII) in 1963, but added a few new provisions. The Treaty was opened for signature by the three depository Governments (the Russian Federation, the United Kingdom and the United States of America) in January 1967, and it entered into force in October 1967. The Outer Space Treaty provides the basic framework on international space law, including the following principles:

• the exploration and use of outer space shall be carried out for the benefit and in the interests of all countries and shall be the province of all mankind;

• outer space shall be free for exploration and use by all States;

• outer space is not subject to national appropriation by claim of sovereignty, by means of use or occupation, or by any other means; o States shall not place nuclear weapons or other weapons of mass destruction in orbit or on celestial bodies or station them in outer space in any other manner;

• the Moon and other celestial bodies shall be used exclusively for peaceful purposes;

• astronauts shall be regarded as the envoys of mankind; o States shall be responsible for national space activities whether carried out by governmental or non-governmental entities; o States shall be liable for damage caused by their space objects; and www.insightsonindia.com 27 INSTA Revision 3.0 INSIGHTSIAS SIMPLYFYING IAS EXAM PREPARATION

o States shall avoid harmful contamination of space and celestial bodies.

• The Outer Space Treaty prohibits only weapons of mass destruction in outer space, not ordinary weapons.

• As of February 2019, 108 countries are parties to the treaty, while another 23 have signed the treaty but have not completed ratification. https://www.thehindu.com/opinion/editorial/outer-clarity/article26737897.ece https://www.unoosa.org/oosa/en/ourwork/spacelaw/treaties/introouterspacetrea ty.html

36. Consider the following statements regarding rainfall in India 1. Monsoons winds flowing in Rajasthan and Gujarat are not obstructed by any orographic barrier and hence these regions receive no rainfall. 2. Cherrapunji and Mawsynram receive abnormally high rainfall due to funneling effect followed by orographic upliftment. Which of the statements given above is/are correct? (a) 1 only (b) 2 only (c) Both 1 and 2 (d) Neither 1 nor 2

Solution: C

Why no significant rainfall in Gujarat and Rajasthan? Explain the formation of Thar Desert?

• Monsoons winds flowing in Rajasthan and Gujarat are not obstructed by any orographic barrier and hence these regions receive no rainfall. [Monsoon winds blow almost parallel to Aravalis and hence there is no orographic rainfall]. [No convection cell or vertical wind movements arise in Rajasthan and Gujarat: Monsoon winds blow towards low pressure cells in Tibet and hence only horizontal wind movements exist in Gujarat and Rajasthan] [Sub-tropical high pressure belt: In winter the region experiences strong divergence because of the STJ – Sub-Tropical Jet.] How come Cherrapunji and Mawsynram receive abnormally high rainfall?

• Mawsynram and Cherrapunji are the wettest places on earth with mean annual rainfall over 1000 cm.

www.insightsonindia.com 28 INSTA Revision 3.0 INSIGHTSIAS SIMPLYFYING IAS EXAM PREPARATION

• Copious rainfall in these places is due to funneling effect followed by orographic upliftment. [Funneling effect = clouds are channeled into a narrow region between mountains and hence the cloud density is extraordinary]

37. Consider the following statements regarding Easterly Jet Stream 1. This jet stream is responsible for bringing western disturbances from the Mediterranean region in to the Indian sub-continent. 2. This helps in the sudden onset of the south-west monsoons. Which of the statements given above is/are correct? (a) 1 only (b) 2 only (c) Both 1 and 2 (d) Neither 1 nor 2

Solution: B

Westerly Jet Stream

• Westerly jet stream blows at a very high-speed during winter over the sub- tropical zone.

• Southern branch of the jet stream exercises a significant influence on the winter weather conditions in India.

• This jet stream is responsible for bringing western disturbances from the Mediterranean region in to the Indian sub-continent.

• Winter rain and heat storms in north-western plains and occasional heavy snowfall in hilly regions are caused by these disturbances.

• These are generally followed by cold waves in the whole of northern plains. Easterly Jet Stream

• Reversal in upper air circulation takes place in summer due to the apparent shift of the sun’s vertical rays in the northern hemisphere.

• The westerly jet stream is replaced by the easterly jet stream which owes its origin to the heating of the Tibet plateau.

• This helps in the sudden onset of the south-west monsoons.

www.insightsonindia.com 29 INSTA Revision 3.0 INSIGHTSIAS SIMPLYFYING IAS EXAM PREPARATION

38. Consider the following statements regarding the Pradhan Mantri Sahaj Bijli Har Ghar Yojana (Saubhagya): 1. The objective of the scheme is to provide energy access to all by last mile connectivity. 2. The beneficiaries for free electricity connections would be identified using Socio Economic and Caste Census (SECC) 2011 data. 3. The Rural Electrification Corporation Limited (REC) will remain the nodal agency for the operationalisation of the scheme throughout the country. Which of the statements given above is/are correct? (a) 1 and 2 only (b) 2 and 3 only (c) 1 and 3 only (d) 1, 2 and 3

Solution: D

Pradhan Mantri Sahaj Bijli Har Ghar Yojana –“Saubhagya” is a scheme to ensure electrification of all willing households in the country in rural as well as urban areas. The objective of the ‘Saubhagya’ is to provide energy access to all by last mile connectivity and electricity connections to all remaining un-electrified households in rural as well as urban areas to achieve universal household electrification in the country. The beneficiaries for free electricity connections would be identified using Socio Economic and Caste Census (SECC) 2011 data. However, un-electrified households not covered under the SECC data would also be provided electricity connections under the scheme on payment of Rs. 500 which shall be recovered by DISCOMs in 10 instalments through electricity bill. The solar power packs of 200 to 300 Wp with battery bank for un-electrified households located in remote and inaccessible areas, comprises of Five LED lights, One DC fan, One DC power plug. It also includes the Repair and Maintenance (R&M) for 5 years. The Rural Electrification Corporation Limited (REC) will remain the nodal agency for the operationalisation of the scheme throughout the country. https://www.thehindu.com/opinion/op-ed/how-to-achieve-24×7-power-for- all/article26714432.ece https://vikaspedia.in/energy/policy-support/pradhan-mantri-sahaj-bijli-har-ghar- yojana

www.insightsonindia.com 30 INSTA Revision 3.0 INSIGHTSIAS SIMPLYFYING IAS EXAM PREPARATION

39. Consider the following statements regarding Red Soils 1. These soils mostly occur in the regions of low rainfall. 2. They are poor in lime, magnesia, phosphates, nitrogen and humus. 3. They are basic mainly due to the nature of the parent rocks. Which of the statements given above is/are correct? (a) 1 and 2 only (b) 1 and 3 only (c) 2 and 3 only (d) 1, 2 and 3

Solution: A

Red Soils

• Red soils along with its minor groups form the largest soil group of India.

• The main parent rocks are crystalline and metamorphic rocks like acid granites, gneisses and quartzites. Characteristics of Red Soils

• The texture of these soils can vary from sand to clay, the majority being loams.

• On the uplands, the red soils are poor, gravelly, and porous. But in the lower areas they are rich, deep dark and fertile. Chemical Composition of Red Soils

• They are acidic mainly due to the nature of the parent rocks. The alkali content is fair.

• They are poor in lime, magnesia, phosphates, nitrogen and humus.

• They are fairly rich in potash and potassium. Distribution of Red Soils

• These soils mostly occur in the regions of low rainfall.

• They occupy about 3.5 lakh sq km (10.6 per cent) of the total area of the country.

40. Consider the following statements regarding the Indian Agriculture: 1. India is the largest producer of spices. 2. Agricultural export constitutes around 10 per cent of the country’s exports. 3. India is the second largest producer of sugar. www.insightsonindia.com 31 INSTA Revision 3.0 INSIGHTSIAS SIMPLYFYING IAS EXAM PREPARATION

Which of the statements given above is/are correct? (a) 1 and 2 only (b) 2 and 3 only (c) 1 and 3 only (d) 1, 2 and 3

Solution: D

Agriculture, along with fisheries and forestry, is one of the largest contributors to the Gross Domestic Product. Gross Value Added by agriculture, forestry and fishing is estimated at Rs 17.67 trillion (US$ 274.23 billion) in FY18. As per estimates by the Central Statistics Office (CSO), the share of agriculture and allied sectors (including agriculture, livestock, forestry and fishery) was 15.35 per cent of the Gross Value Added (GVA) during 2015-16. India is the largest producer, consumer and exporter of spices and spice products. Agricultural export constitutes 10 per cent of the country’s exports and is the fourth- largest exported principal commodity. India, the second-largest producer of sugar, accounts for 14 per cent of the global output. It is the sixth-largest exporter of sugar, accounting for 2.76 per cent of the global exports. India is the leading producer of wheat, rice, cotton, sugarcane & vegetables. https://www.financialexpress.com/opinion/agriculture-needs-a-shift-in-focus-ten- ways-india-can-do-it/1542175/

41. Consider the following statements regarding Forest/Mountain Soils 1. The formation of these soils is mainly governed by the characteristic deposition of organic matter derived from forests. 2. Their character changes with parent rocks, ground-configuration and climate, hence they differ greatly even if they occur in close proximity to one another. 3. They are rich in potash, phosphorus and lime. Which of the statements given above is/are correct? (a) 1 and 2 only (b) 1 and 3 only (c) 2 and 3 only www.insightsonindia.com 32 INSTA Revision 3.0 INSIGHTSIAS SIMPLYFYING IAS EXAM PREPARATION

(d) 1, 2 and 3

Solution: A

Forest – Mountain Soils

• These soils occupy about 2.85 lakh sq km or 8.67% of the total land area of India.

• They are mainly heterogeneous soils found on the hill slopes covered by forests.

• The formation of these soils is mainly governed by the characteristic deposition of organic matter derived from forests and their character changes with parent rocks, ground-configuration and climate.

• Consequently, they differ greatly even if they occur in close proximity to one another. Distribution of Forest – Mountain Soils

• In the Himalayan region, such soils are mainly found in valleys, less steep and north facing slopes. The south facing slopes are very steep and exposed to denudation and hence do not support soil formation.

• Forest soils occur in Western and Eastern Ghats also. Chemical properties of Forest – Mountain Soils

• The forest soils are very rich in humus.

• They are deficient in potash, phosphorus and lime.

• They require good deal of fertilizers for high yields.

42. Which of the following measures control soil erosion in India? 1. Ban on shifting cultivation 2. Controlled grazing 3. Mixed cropping Select the correct answer using the code given below (a) 2 only (b) 1 and 3 only (c) 1, 2 and 3 (d) 2 and 3 only

Solution: C

www.insightsonindia.com 33 INSTA Revision 3.0 INSIGHTSIAS SIMPLYFYING IAS EXAM PREPARATION

Soil conservation is the prevention of loss of the top most layer of the soil from erosion or prevention of reduced fertility caused by over usage, acidification, salinization or other chemical soil contamination. Contour tillage: Here the fields are ploughed along the contours and not along the hill slope. This type of plantation is very effective as it leads to the formation of ridges and furrows against the direction of flow and decreases the velocity of water. Contour Bunding: Here first the plants and bushes in the plot of shifting cultivation is cut down. Leaves are slashed down and left on the surface of the soil to get dry. After the decomposition of the leaves and various other plant parts, the dry materials are bundled and arranged in contour lines. Some other measures to control soil erosion are: construction of check dams, ban on shifting cultivation, controlled grazing, afforestation, mixed cropping and mixed farming etc. Planting species of plants that can restore the ecological balance of an eroded area is also necessary as rehabilitation of the damaged soil is as much important as its prevention from getting eroded.

43. Chandaka-Dampara Wildlife Sanctuary, is located in: (a) (b) Odisha (c) Manipur (d) Madhya Pradesh

Solution: B

Chandaka Sanctuary is a wildlife reserve located in north western fringe of Bhubaneswar in the Indian state of Odisha. Chandaka –Dampara Wildlife Sanctuary comprises major forest area i.e. notified forest blocks of Chandaka Wildlife Division. Four decades later, bamboo rice shows up in Odisha. The rare variety, which was last harvested in Chandaka-Dampara Wildlife Sanctuary in 1979, grows twice or thrice in a century. What is Bamboo Rice? Bamboo rice is special rice that is grown out of a dying bamboo shoot. When the bamboo shoot breathes its last, it flowers into a rare variety of rice seeds, which are known as bamboo rice.

44. Which of the following forces/factors affects the velocity and direction of Wind? 1. Pressure gradient force www.insightsonindia.com 34 INSTA Revision 3.0 INSIGHTSIAS SIMPLYFYING IAS EXAM PREPARATION

2. Coriolis force 3. Gravitational force Select the correct answer using the code given below: (a) 1, 2 and 3 (b) 2 and 3 only (c) 1 and 2 only (d) 1 and 3 only

Solution: A

Forces Affecting the Velocity and Direction of Wind

• The air is set in motion due to the differences in atmospheric pressure. The air in motion is called wind. The wind blows from high pressure to low pressure. The wind at the surface experiences friction. In addition, rotation of the earth also affects the wind movement.

• The force exerted by the rotation of the earth is known as the Coriolis force. Thus, the horizontal winds near the earth surface respond to the combined effect of three forces – the pressure gradient force, the frictional force and the Coriolis force. In addition, the gravitational force acts downward.

45. Bold Kurukshetra – 2019, is a joint military exercise between: (a) India – Bangladesh (b) India – Bolivia (c) India – Singapore (d) India –

Solution: C

The 12th edition of joint military exercise between India and Singapore, BOLD KURUKSHETRA 2019, culminated on 11 April 2019 after an impressive closing ceremony held at Babina Military Station. The four day long joint training focused on developing interoperability and conduct of joint tactical operations in mechanised warfare.

www.insightsonindia.com 35 INSTA Revision 3.0 INSIGHTSIAS SIMPLYFYING IAS EXAM PREPARATION

46. Consider the following statements regarding Western disturbances in Northern Plains 1. These are low-pressure systems, originate over the Mediterranean Sea and Western Asia 2. They are of immense importance for the cultivation of ‘rabi’ crops. Which of the statements given above is/are correct? (a) 1 only (b) 2 only (c) Both 1 and 2 (d) Neither 1 nor 2

Solution: C

A characteristic feature of the cold weather season over the northern plains is the inflow of cyclonic disturbances from the west and the northwest. These low-pressure systems, originate over the Mediterranean Sea and western Asia and move into India, along with the westerly flow. They cause the much-needed winter rains over the plains and snowfall in the mountains. Although the total amount of winter rainfall locally known as ‘mahawat’ is small, they are of immense importance for the cultivation of ‘rabi’ crops.

47. Consider the following statements regarding El-Nino and La-Nina 1. El Nino and La Nina episodes typically last nine to 12 months. 2. El Nino and La Nina events occur on average every two to seven years. Which of the statements given above is/are not correct? (a) 1 only (b) 2 only (c) Both 1 and 2 (d) Neither 1 nor 2

Solution: D

La Niña is sometimes referred to as the cold phase of ENSO and El Niño as the warm phase of ENSO. These deviations from normal surface temperatures can have large-scale impacts not only on ocean processes, but also on global weather and climate. www.insightsonindia.com 36 INSTA Revision 3.0 INSIGHTSIAS SIMPLYFYING IAS EXAM PREPARATION

El Niño and La Niña episodes typically last 9 to 12 months, but some prolonged events may last for years. While their frequency can be quite irregular, El Niño and La Niña events occur on average every two to seven years. Typically, El Niño occurs more frequently than La Niña. https://oceanservice.noaa.gov/facts/ninonina.html

48. Consider the following statements regarding the International Maritime Organisation (IMO): 1. It is a specialized agency of the United Nations. 2. It is the global standard-setting authority for the safety, security and environmental performance of international shipping. Which of the statements given above is/are correct? (a) 1 only (b) 2 only (c) Both 1 and 2 (d) Neither 1 nor 2

Solution: C

As a specialized agency of the United Nations, IMO is the global standard-setting authority for the safety, security and environmental performance of international shipping. Its main role is to create a regulatory framework for the shipping industry that is fair and effective, universally adopted and universally implemented. In other words, its role is to create a level playing-field so that ship operators cannot address their financial issues by simply cutting corners and compromising on safety, security and environmental performance. This approach also encourages innovation and efficiency. https://www.thehindubusinessline.com/economy/logistics/imos-new-rule-on- electronic-information-exchange-between-ships-and-ports-comes-into- force/article26771859.ece http://www.imo.org/en/About/Pages/Default.aspx

49. Consider the following statements regarding the Temperate Cyclone 1. Temperate cyclone is largely of thermal origin. 2. Temperate cyclone can form both on land as well as seas. 3. Temperate cyclones typically won’t last for more than one week. Which of the statements given above is/are correct? www.insightsonindia.com 37 INSTA Revision 3.0 INSIGHTSIAS SIMPLYFYING IAS EXAM PREPARATION

(a) 2 only (b) 1 only (c) 1 and 3 only (d) 1 and 2 only

Solution: A

A tropical cyclone is a rapidly rotating storm system characterized by a low- pressure center, a closed low-level atmospheric circulation, strong winds, and a spiral arrangement of thunderstorms that produce heavy rain or squalls. Tropical cyclone is largely of thermal origin. Tropic cyclone can form only on seas.

Temperate Cyclones or Extra tropical cyclones, sometimes called mid-latitude cyclones or wave cyclones, are low-pressure areas which, along with the anticyclones of high-pressure areas, drive the weather over much of the Earth. Temperate cyclone can form both on land as well as seas. Temperate cyclones typically last for 2-3 weeks.

50. Recently, on which of the following rivers ‘Maitri Bridge’ was constructed by ? (a) Jhelum (b) Ravi (c) Indus (d) Brahmaputra

Solution: C

Indian Army built the longest suspension bridge over in Leh in just 40 days, making a record of its own. Named Maitri Bridge, the construction of the bridge was undertaken by the combat engineers of ‘Sahas aur Yogyata’ regiment of ‘Fire and Fury Corps’ of the Indian Army. The bridge is built using around 500 tonnes of bridge equipment and construction material. The 260-feet long bridge is likely to give a major boost to connectivity in remote areas in Ladakh. The cable suspension bridge that has been opened for the public was constructed to help the residents of Choglamsar, Stok and Chuchot villages https://www.businesstoday.in/current/economy-politics/longest-suspension- bridge-over-indus-river-opens-to-public-heres-all-you-need-to- know/story/333520.html www.insightsonindia.com 38 INSTA Revision 3.0 INSIGHTSIAS SIMPLYFYING IAS EXAM PREPARATION

DAY – 15

51. Consider the following statements regarding drainage patterns 1. In a concordant drainage pattern, the river follows its initial path irrespective of the changes in topography. 2. In a discordant drainage pattern, the path of the river is highly dependent on the slope of the river and topography. Which of the statements given above is/are correct? (a) 1 only (b) 2 only (c) Both 1 and 2 (d) Neither 1 nor 2

Solution: D

Concordant drainage patterns

• A drainage pattern is described as concordant if it correlates to the topology and geology of the area.

• In simple words, in a concordant drainage pattern, the path of the river is highly dependent on the slope of the river and topography.

• Concordant drainage patterns are the most commonly found drainage patterns and are classified into many consequent, subsequent, obsequent and resequent. Discordant or Insequent drainage patterns

• A drainage pattern is described as discordant if it does not correlate to the topology (surface relief features) and geology of the area.

• In simple words, in a discordant drainage pattern, the river follows its initial path irrespective of the changes in topography.

• Discordant drainage patterns are classified into two main types: antecedent and superimposed.

• Usually, rivers in both these drainage types flow through a highly sloping surface.

52. Consider the following statements regarding Damodar River 1. It rises in the Amarkantak Plateau. 2. It flows through a rift valley. www.insightsonindia.com 39 INSTA Revision 3.0 INSIGHTSIAS SIMPLYFYING IAS EXAM PREPARATION

3. It is called as ‘Sorrow of Bengal’. Which of the statements given above is/are correct? (a) 1 and 2 only (b) 1 and 3 only (c) 2 and 3 only (d) 1, 2 and 3

Solution: C

Son River

• The Son River rises in the Amarkantak Plateau.

• Its source is close to the origin of the Narmada.

• It passes along the Kaimur Range.

• It joins the Ganga near Danapur in Patna district of Bihar.

• It flows for a distance of 784 km from its source.

• The important tributaries of the Son are the Johilla, the Gopat, the Rihand, the Kanhar and the North Koel. Almost all the tributaries join it on its right bank.

Damodar river

• The Damodar river rises in the hills of the Chotanagpur plateau and flows through a rift valley.

• Rich in mineral resources, the valley is home to large-scale mining and industrial activity.

• It has a number of tributaries and subtributaries, such as Barakar, Konar, Bokaro, Haharo, etc.

• The Barakar is the most important tributary of the Damodar.

• Several dams have been constructed in the valley, for the generation of hydroelectric power. The valley is called “the Ruhr of India”.

• The first dam was built across the Barakar River, a tributary of the Damodar river.

• It used to cause devastating floods as a result of which it earned the name ‘Sorrow of Bengal’. Now the river is tamed by constructing numerous dams.

www.insightsonindia.com 40 INSTA Revision 3.0 INSIGHTSIAS SIMPLYFYING IAS EXAM PREPARATION

53. Hala’ib triangle, a disputed territory, sometimes seen in news, is located on the coast of: (a) Persian Gulf (b) Gulf of Oman (c) Red Sea (d) South China Sea

Solution: C

The Halayib Triangle, also known as the Halayeb Triangle, is an area of land measuring 20,580 square kilometres located on the Northeast African coast of the Red Sea. It is a disputed territory between Sudan and Egypt.

54. Consider the following statements regarding Peninsular River System 1. Peninsula rivers are much older than the Himalayan rivers and have reached mature stage. 2. The peninsular drainage is mainly Discordant. 3. The main water divide in peninsular rivers is formed by the Western Ghats. Which of the statements given above is/are correct? (a) 1 and 2 only (b) 1 and 3 only (c) 2 and 3 only (d) 1, 2 and 3

Solution: B

Peninsular River System or Peninsular Drainage

• Peninsula rivers are much older than the Himalayan rivers {Discordant}.

• The peninsular drainage is mainly Concordant except for few rivers in the upper peninsula region.

• They are non-perennial rivers with a maximum discharge in the rainy season.

• The peninsular rivers have reached mature stage {Fluvial Landforms} and have almost reached their base level. [Vertical downcutting is negligible]. www.insightsonindia.com 41 INSTA Revision 3.0 INSIGHTSIAS SIMPLYFYING IAS EXAM PREPARATION

• The rivers are characterized by broad and shallow valleys.

• The river banks have gentle slopes except for a limited tract where faulting forms steep sides.

• The main water divide in peninsular rivers is formed by the Western Ghats, which run from north to south close to the western coast.

• The velocity of water in the rivers and the load carrying capacity of the streams is low due to low gradient.

• Most of the major rivers of the peninsula such as the Mahanadi, the Godavari, the Krishna and the Cauvery flow eastwards and drain into the Bay of Bengal. These rivers make deltas at their mouths.

• But the west flowing rivers of Narmada and Tapi as well as those originating from the Western Ghats and falling in the Arabian Sea form estuaries in place of deltas.

55. Consider the following pairs of Harvest festivals in India and states they are associated with: Harvest Festivals States 1. Vishu - Maharashtra 2. Rongali Bihu - Odisha 3. Puthandu Pirappu - Which of the pairs given above is/are correctly matched? (a) 1 and 3 only (b) 2 only (c) 1, 2 and 3 (d) None

Solution: D

Vishu is a Hindu festival celebrated in the Indian state of Kerala, Tulu Nadu region in Karnataka, Mahe district of Union Territory of Pondicherry, neighbouring areas of Tamil Nadu and their diaspora communities. The festival marks the first day of Medam, the ninth month in the solar calendar followed in Kerala. Bohag Bihu or Rongali Bihu also called Xaat Bihu is a festival celebrated in the Indian state of Assam and other parts of northeastern India, and marks the beginning of the Assamese New Year. It usually falls in the 2nd week of April, historically signifying the time of harvest. Puthandu, also known as Puthuvarusham or Tamil New Year, is the first day of year on the Tamil calendar and traditionally celebrated as a festival. The festival date is set with the solar cycle of the lunisolar Hindu calendar, as the first day of the Tamil month Chithirai. www.insightsonindia.com 42 INSTA Revision 3.0 INSIGHTSIAS SIMPLYFYING IAS EXAM PREPARATION

56. Consider the following statements regarding Jhelum River 1. The Jhelum originates from near the Bara Lacha Pass in the Lahul-Spiti part of the Zaskar Range. 2. It flows northwards into Wular Lake. Which of the statements given above is/are correct? (a) 1 only (b) 2 only (c) Both 1 and 2 (d) Neither 1 nor 2

Solution: B

Jhelum River

• The Jhelum has its source in a spring at Verinag in the south-eastern part of the Kashmir Valley.

• It flows northwards into Wular Lake (north-western part of Kashmir Valley). From Wular Lake, it changes its course southwards. At Baramulla the river enters a gorge in the hills.

• The river forms steep-sided narrow gorge through below Baramula.

• At Muzaffarabad, the river takes a sharp hairpin bend southward.

• Thereafter, it forms the India- boundary for 170 km and emerges at the Potwar Plateau near Mirpur.

• After flowing through the spurs of the Salt Range it debouches (emerge from a confined space into a wide, open area) on the plains near the city of Jhelum.

• It joins the Chenab at Trimmu.

• The river is navigable for about 160 km out of a total length of 724 km.

Chenab River

• The Chenab originates from near the Bara Lacha Pass in the Lahul-Spiti part of the Zaskar Range.

57. Which of the following are left bank tributaries of The Ganga River? 1. Ramganga River 2. Yamuna River www.insightsonindia.com 43 INSTA Revision 3.0 INSIGHTSIAS SIMPLYFYING IAS EXAM PREPARATION

3. Damodar River 4. Gandak River Select the correct answer using the code given below: (a) 1 and 2 only (b) 1 and 4 only (c) 1, 2 and 4 only (d) 1, 2, 3 and 4

Solution: B

Left Bank Tributaries of the Ganga River

• Ramganga River

• Ghaghra River

• Kali River

• Gandak River

• Burhi Gandak

• Kosi River Right Bank Tributaries of the Ganga

• Yamuna River

• Damodar River

58. Consider the following statements regarding the Drugs Technical Advisory Board (DTAB): 1. It is a non-statutory body, set up by executive resolution. 2. It is part of Central Drugs Standard Control Organization (CDSCO). Which of the statements given above is/are correct? (a) 1 only (b) 2 only (c) Both 1 and 2 (d) Neither 1 nor 2

Solution: B www.insightsonindia.com 44 INSTA Revision 3.0 INSIGHTSIAS SIMPLYFYING IAS EXAM PREPARATION

DTAB is highest statutory decision-making body on technical matters related to drugs in the country. It is constituted as per the Drugs and Cosmetics Act, 1940. It is part of Central Drugs Standard Control Organization (CDSCO) in the Ministry of Health and Family Welfare. https://www.thehindu.com/sci-tech/health/government-notifies-medical-devices- as-drugs-industry-cautious/article30812704.ece

59. Consider the following statements regarding Glaciers 1. The Siachen Glacier in Nubra valley is the largest glacier outside the polar and the sub-polar regions. 2. The glaciers of the Pir Panjal Range are less numerous and smaller in size as compared to those of the Range. 3. Gangotri Glacier is located in Uttarkashi, Uttarakhand. Which of the statements given above is/are correct? (a) 1 and 2 only (b) 1 and 3 only (c) 2 and 3 only (d) 1, 2 and 3

Solution: D

Glaciers of the Karakoram Range

• Maximum development of glaciers occurs in the Karakoram range.

• Some of the largest glaciers outside the polar and sub-polar regions are found in this range. The southern side of this range has many gigantic glaciers.

• The 75 km long Siachen Glacier in Nubra valley has the distinction of being the largest glacier outside the polar and the sub-polar regions.

• The second largest is the 74 km long Fedchenko Glacier (Pamirs)

• Third largest is the Hispar Glacier. It is 62 km long and occupies a tributary of the Hunza River.

Glaciers of the Pir Panjal Range

• The glaciers of the Pir Panjal Range are less numerous and smaller in size as compared to those of the Karakoram Range.

• The longest Sonapani Glacier in the Chandra Valley of Lahul and Spiti region is only 15 km long. www.insightsonindia.com 45 INSTA Revision 3.0 INSIGHTSIAS SIMPLYFYING IAS EXAM PREPARATION

• Gangotri Glacier: located in Uttarkashi, Uttarakhand

• Milam Glacier: located in Uttarkashi, Uttarakhand

• Pindari glacier Nanda devi, Uttarakhand

• Zemu Glacier Largest glacier in the Eastern Himalaya Located on Kanchenjunga peak, Sikkim

• Baltoro Glacier J & K – Karakoram range

• Chogolungma glacier J & K – Karakoram range

• Diamir Glacier J & K – Karakoram range

• Siachin Glacier LOC between India & POK; Close to China border; – Karakoram Range

60. Consider the following statements regarding the Asia Pacific Broadcasting Union (ABU): 1. It was established as a non-profit, non-governmental, non- political, professional association. 2. It stipulates that there may be only five Full Members in any one country. 3. It is the biggest broadcasting union in the world. Which of the statements given above is/are correct? (a) 1 and 2 only (b) 2 and 3 only (c) 1 and 3 only (d) 1, 2 and 3

Solution: C

The Asia-Pacific Broadcasting Union (ABU) is the biggest broadcasting union in the world. The Union was established in 1964 as a non-profit, non-governmental, non- political, professional association with mandate to assist the development of broadcasting in the region. ABU promotes the collective interests of television and radio broadcasters as well as key industry players and facilitate regional and international media co-operation. ABU is a member of the World Broadcasters’ Union and works closely with the other regional broadcasting unions on matters of common concern such as reserving frequencies for broadcasters, harmonisation of operating and technical broadcasting standards and systems and finalising the Broadcasting Treaty.

www.insightsonindia.com 46 INSTA Revision 3.0 INSIGHTSIAS SIMPLYFYING IAS EXAM PREPARATION

Membership is open to national broadcasters and national broadcasting organisations. There are only two Full members in each country so that there is a balance of voting rights across the region. The ABU’s “board of directors” is called the Administrative Council. It consists of 17 of its Full and Additional Full members, elected for a three year term. http://www.newsonair.com/News?title=ABU-Media-Summit-on-Climate-Action- and-Disaster-Preparedness-underway-in-Nepal&id=362714

61. Consider the following statements regarding Biomass 1. The energy from biomass is released on burning or breaking the chemical bonds of organic molecules formed during photosynthesis. 2. Biomass fuels can be used directly. Which of the statements given above is/are correct? (a) 1 only (b) 2 only (c) Both 1 and 2 (d) Neither 1 nor 2

Solution: C

Biomass [Conventional Source]

• Biomass is a renewable energy resource derived from plant and animal waste.

• The energy from biomass (biomass conversion) is released on burning or breaking the chemical bonds of organic molecules formed during photosynthesis.

• Biomass fuels can be used directly or they can be transformed into more convenient form and then used. Sources of biomass

• By-products from the timber industry, agricultural crops and their byproducts, raw material from the forest, major parts of household waste and wood.

• Solid Biomass fuels: Wood logs and wood pellets, charcoal, agricultural waste (stalks and other plant debris), animal waste (dung), aquatic plants (kelp and water hyacinths) urban waste (paper, cardboard and other combustible materials).

62. Consider the following statements regarding Graphite 1. Graphite is a naturally-occurring form of crystalline carbon. www.insightsonindia.com 47 INSTA Revision 3.0 INSIGHTSIAS SIMPLYFYING IAS EXAM PREPARATION

2. Graphite is a non-metal and it is the only non-metal that can conduct electricity. 3. Graphite is hard and cleaves with very high pressure. Which of the statements given above is/are correct? (a) 1 and 2 only (b) 1 and 3 only (c) 2 and 3 only (d) 1, 2 and 3

Solution: A

Graphite

• Graphite is a naturally-occurring form of crystalline carbon.

• It is also known as plumbago or black lead.

• The carbon content in Graphite is never less than 95%.

• Graphite may be considered the highest grade of coal, just above anthracite. Carbon content in Peat < Lignite < Bituminous < Anthracite < Graphite < Diamond

• It is not normally used as fuel because it is difficult to ignite.

• It is found in metamorphic and igneous rocks.

• Graphite is extremely soft, cleaves [splits into layers] with very light pressure.

• It is extremely resistant to heat and is highly unreactive.

• Most of the graphite is formed at convergent plate boundaries where organic- rich shales and limestones were subjected to metamorphism due to heat and pressure.

• Metamorphism produces marble, schist and gneiss that contains tiny crystals and flakes of graphite.

• Some graphite forms from the metamorphism of coal seams. This graphite is known as “amorphous graphite”.

• Graphite is a non-metal and it is the only non-metal that can conduct electricity.

63. The ‘Kunming Initiative’, sometimes seen in news is related to: (a) The South Asian Free Trade Area (SAFTA) (b) The International North–South Transport Corridor (INSTC) www.insightsonindia.com 48 INSTA Revision 3.0 INSIGHTSIAS SIMPLYFYING IAS EXAM PREPARATION

(c) Bangladesh, China, India and Myanmar (BCIM) economic corridor (d) East Asia Summit (EAS)

Solution: C

The four member countries of the Kunming Initiative that was rechristened as the BCIM Forum (Bangladesh, India, China, and Myanmar) for regional cooperation is a Track II initiative that was given Track I coordination in 2011. The Kunming Initiative was formed keeping in mind the fact that a regional outlook on the concerns of development, security and prosperity was undoubtedly more beneficial than striking out one’s own. https://www.business-standard.com/article/news-ians/china-drops-bcim-from- bri-projects-list-119042800540_1.html

64. Which of the following are the coalfields in Jharkhand? 1. 2. 3. Rajmahal coalfield 4. Talcher field Select the correct answer using the code given below: (a) 1 and 2 only (b) 2 and 3 only (c) 2 and 4 only (d) 1 and 4 only

Solution: B

Gondwana Coalfields in Chhattisgarh

• Korba coalfield • Jhilmili coalfield • Birampur coalfield • Johilla coalfield • Hasdo-Arand coalfield • Sonhat coalfield • Chirmiri coalfield • Tatapani-Ramkota coalfields • Lakhanpur coalfield

www.insightsonindia.com 49 INSTA Revision 3.0 INSIGHTSIAS SIMPLYFYING IAS EXAM PREPARATION

Gondwana Coalfields in Jharkhand

• Jharia coalfield • Karanpura and Ramgarh coalfields • Jayanti coalfields • Auranga coalfield: Palamu district • Bokaro coalfield • Hutar coalfield • West Bokaro [900 m deep] • Deltenganj coalfield • East Bokaro [600 m deep]: • Devgarh coalfields: Dumka district Hazaribagh district • Rajmahal coalfield, Rajmahal hills • Girdih (Karharbari) coalfield

Gondwana Coalfields in Odisha

• Talcher field: Talcher town to Rairkhol in Dhenkanal and Sambalpur districts

• Rampur-Himgir coalfield: Sambalpur and Sundargarh

• Ib river coalfield: Sambalpur and Jharsuguda district

Gondwana Coalfields in Madhya Pradesh

• Singrauli (Waidhian) coalfield: Sidhi and Shandol districts

• Pench-Kanhan-Tawa: Chhindwara district

: Shandol district

: Umaria district

65. Consider the following statements regarding the Malaria disease: 1. It is caused by Plasmodium parasites. 2. It is preventable and curable disease. Which of the statements given above is/are correct? (a) 1 only (b) 2 only (c) Both 1 and 2 (d) Neither 1 nor 2

Solution: C

Malaria is caused by Plasmodium parasites. The parasites are spread to people through the bites of infected female Anopheles mosquitoes, called “malaria vectors.” There are 5 parasite species that cause malaria in humans, and 2 of these species – P. falciparum and P. vivax – pose the greatest threat. www.insightsonindia.com 50 INSTA Revision 3.0 INSIGHTSIAS SIMPLYFYING IAS EXAM PREPARATION

It is preventable and curable. https://www.who.int/news-room/fact-sheets/detail/malaria https://www.businesstoday.in/pti-feed/icmr-launches-mera-india-to-eliminate- malaria-from-india-by-2030/story/340653.html

66. Which of the following places has/have copper reserves 1. Balaghat district, Madhya Pradesh 2. Khetri-Singhana belt in Rajasthan 3. Singhbhum, Jharkhand Select the correct answer using the code given below: (a) 1 and 2 only (b) 1 and 3 only (c) 2 and 3 only (d) 1, 2 and 3

Solution: D

Copper Reserves in India- 46 million tonnes.

• Rajasthan (50%)

• Madhya Pradesh (24%)

• Jharkhand (19%) The rest 7 per cent in AP, Gujarat, Haryana, Karnataka etc.

Madhya Pradesh

• 1st in production [59.85 %].

• Malanjkhand copper mines of Balaghat district are the most important ones.

• Reserves of moderate size are also found in Betul district. Rajasthan

• 2nd in production [28%]

• Found along the Aravali range.

• Ajmer, Alwar, Bhilwara, Chittaurgarh, Dungarpur, Jaipur, Jhunjhunu, Pali, Sikar, Sirohi and Udaipur districts.

www.insightsonindia.com 51 INSTA Revision 3.0 INSIGHTSIAS SIMPLYFYING IAS EXAM PREPARATION

• Khetri-Singhana belt in Jhunjhunu district is the most important copper producing area. Jharkhand

• 3rd in production [11 %].

• Singhbhum is the most important copper producing district.

• Found in Hazaribagh district, Santhal Parganas and Palamu districts.

67. Zemu Glacier is located in the State of: (a) Sikkim (b) Arunachal Pradesh (c) Himachal Pradesh (d) Uttarakhan

Solution: A

Zemu Glacier is the largest glacier in the Eastern Himalaya. It is about 26 kilometres in length and is located at the base of Kangchenjunga in the Himalayan region of Sikkim, India. The Zemu Glacier drains the east side of Kanchenjunga, the world’s third highest mountain.

68. Manjeera Wildlife sanctuary, sometimes seen in news, located in the state of: (a) Telangana (b) Andhra Pradesh (c) Karnataka (d) Maharashtra

Solution: A

Manjira wildlife sanctuary is a wildlife sanctuary and reservoir located in Sangareddy district of Telangana State, India. Originally a crocodile sanctuary, today more than 70 species of birds are spotted here and is home for the vulnerable species mugger crocodile. https://timesofindia.indiatimes.com/city/hyderabad/as-manjeera-and-singur-dry- up-water-board-falls-back-on-citys-twin-reservoirs/articleshow/69604449.cms

www.insightsonindia.com 52 INSTA Revision 3.0 INSIGHTSIAS SIMPLYFYING IAS EXAM PREPARATION

69. Consider the following statements regarding Polymetallic Nodules 1. Polymetallic nodules are rounded accretions of manganese and iron hydroxides that cover vast areas of the seafloor 2. India signed a 15-year contract for exploration of Polymetallic Nodules (PMN) in the Central Indian Ocean Basin with the International Seabed Authority (ISA) Which of the statements given above is/are correct? (a) 1 only (b) 2 only (c) Both 1 and 2 (d) Neither 1 nor 2

Solution: C

Polymetallic nodules are rounded accretions of manganese and iron hydroxides that cover vast areas of the seafloor, but are most abundant on abyssal plains at water depths of 4000-6500 metres. They form through the aggregation of layers of iron and manganese hydroxides around a central particle (such as a shell or small rock fragment), and range in size from a few millimeters to tens of centimeters. The first deep-sea venture of the National Institute of Oceanography was started in 1982, for the exploration for nodules and India recognized as a “Pioneer Investor”. In 1995, India ratifies the UNCLOS III convention and India elected as the council member of the International Seabed Authority. India signed a 15-year contract for exploration of Polymetallic Nodules (PMN) in the Central Indian Ocean Basin with the International Seabed Authority (ISA) on 25th March, 2002 https://www.eu-midas.net/science/nodules https://www.thehindu.com/sci-tech/science/why-is-india-pulled-to-deep-sea- mining/article28809029.ece

70. Consider the following statements regarding the Bt Brinjal: 1. It was developed by the Indian Council of Agriculture Research (ICAR). 2. It incorporates the cry1Ac gene expressing insecticidal protein. Which of the following statements given above is/are correct? (a) 1 only (b) 2 only (c) Both 1 and 2 www.insightsonindia.com 53 INSTA Revision 3.0 INSIGHTSIAS SIMPLYFYING IAS EXAM PREPARATION

(d) Neither 1 nor 2

Solution: B

Fruit and Shoot Borer (FSB)-resistant brinjal or Bt brinjal was developed using a transformation process similar to the one used in the development of Bt cotton, a biotech crop that was planted on 7.6 million hectares in India in 2008. Bt brinjal incorporates the cry1Ac gene expressing insecticidal protein to confer resistance against FSB. The cry1Ac gene is sourced from the soil bacterium Bacillus thuringiensis (Bt). Bt Brinjal was developed by the Maharashtra Hybrid Seeds Company (Mahyco). The company used a DNA construct containing the cry1Ac gene, a CaMV 35S promoter and the selectable marker genes nptII and aad, to transform young cotyledons of brinjal plants. https://www.thehindubusinessline.com/economy/agri-business/bt-brinjal-being- illegally-grown-in-haryana/article26945204.ece

71. Consider the following pairs Hills State/Region 1. Ajanta Range : Maharashtra 2. Garo Hills : Meghalaya 3. Dafla Hills : Tripura Which of the pairs given above is/are matched correctly? (a) 1 and 3 only (b) 2 only (c) 1 and 2 only (d) 3 only

Solution: C

www.insightsonindia.com 54 INSTA Revision 3.0 INSIGHTSIAS SIMPLYFYING IAS EXAM PREPARATION

72. Consider the following statements 1. India imports coking coal. 2. Anthracite coal is largely found in Jharkhand and Bihar Which of the statements given above is/are correct? (a) 1 only (b) 2 only (c) Both 1 and 2 (d) Neither 1 nor 2

Solution: A

www.insightsonindia.com 55 INSTA Revision 3.0 INSIGHTSIAS SIMPLYFYING IAS EXAM PREPARATION

Coal is originated from organic matter wood. When large tracts of forests are buried under sediments, wood is burnt and decomposed due to heat from below and pressure from above. The phenomenon makes coal but takes centuries to complete. As per the present import policy, coal can be freely imported (under Open General Licence) by the consumers themselves considering their needs based on their commercial prudence. Coking Coal is being imported by Steel Authority of India Limited (SAIL) and other Steel manufacturing units mainly to bridge the gap between the requirement and indigenous availability and to improve the quality of production. Anthracite: It is the best quality of coal with highest calorific value and carries 80 to 95% carbon content. It ignites slowly with a blue flame and found in small quantities in Jammu and Kashmir.

73. Consider the following statements regarding the food additive chemical Melamine: 1. It is high in Nitrogen. 2. It is added to inflate the apparent protein content of food products. 3. There are no approved direct food uses for melamine. Which of the statements given above is/are correct? (a) 1 and 2 only (b) 2 and 3 only (c) 1 and 3 only (d) 1, 2 and 3

Solution: D

Melamine is illegally added to inflate the apparent protein content of food products. Because it is high in nitrogen, the addition of melamine to a food artificially increases the apparent protein content as measured with standard tests. There are no approved direct food uses for melamine, nor are there any recommendations in the Codex Alimentarius. https://www.livemint.com/politics/policy/government-extends-ban-on-import-of- milk-products-from-china-1556036439186.html https://www.who.int/foodsafety/areas_work/chemical-risks/melamine/en/

74. Consider the following statements 1. The Northeastern Plateau extension of the main peninsular plateau. www.insightsonindia.com 56 INSTA Revision 3.0 INSIGHTSIAS SIMPLYFYING IAS EXAM PREPARATION

2. Meghalaya plateau is rich in mineral resources like coal, iron ore, sillimanite, limestone and uranium. 3. Shiwaliks ranges are composed of unconsolidated sediments. Which of the statements given above is/are correct? (a) 1 and 2 only (b) 2 and 3 only (c) 1 and 3 only (d) 1, 2 and 3

Solution: D

The Northeastern Plateau

• In fact it is an extension of the main peninsular plateau. It is believed that due to the force exerted by the northeastward movement of the Indian plate at the time of the Himalayan origin, a huge fault was created between the Rajmahal hills and the Meghalaya plateau.

• Later, this depression got filled up by the deposition activity of the numerous rivers.

• An extension of this is also seen in the Karbi Anglong hills of Assam. Similar to the Chotanagpur plateau, the Meghalaya plateau is also rich in mineral resources like coal, iron ore, sillimanite, limestone and uranium. This area receives maximum rainfall from the south west monsoon.

• The outer-most range of the Himalayas is called the Shiwaliks. They extend over a width of 10-50 Km and have an altitude varying between 900 and 1100 metres. These ranges are composed of unconsolidated sediments brought down by rivers from the main Himalayan ranges located farther north.

75. Consider the following statements regarding the Banking Ombudsman Scheme: 1. The Banking Ombudsman is a senior official appointed by the Reserve Bank of India. 2. It was introduced under the Banking Regulation Act, 1949. 3. All Scheduled Commercial Banks, Regional Rural Banks and Scheduled Primary Co-operative Banks are covered under the Scheme. Which of the statements given above is/are correct? (a) 1 and 2 only (b) 2 and 3 only (c) 1 and 3 only www.insightsonindia.com 57 INSTA Revision 3.0 INSIGHTSIAS SIMPLYFYING IAS EXAM PREPARATION

(d) 1, 2 and 3

Solution: D

The Banking Ombudsman Scheme enables an expeditious and inexpensive forum to bank customers for resolution of complaints relating to certain services rendered by banks. The Banking Ombudsman Scheme is introduced under Section 35 A of the Banking Regulation Act, 1949 by RBI with effect from 1995. The Banking Ombudsman is a senior official appointed by the Reserve Bank of India to redress customer complaints against deficiency in certain banking services. All Scheduled Commercial Banks, Regional Rural Banks and Scheduled Primary Co-operative Banks are covered under the Scheme. https://www.livemint.com/industry/banking/66-of-complaints-mutually-settled- banking-ombudsman-report-1556162402257.html

DAY – 16

76. Consider the following statements regarding PVTG’s 1. Ministry of Tribal Affairs categories tribal groups into Particularly Vulnerable Tribal Groups (PVTG)s. 2. 75 tribal groups have been categorized as Particularly Vulnerable Tribal Groups (PVTG)s across 18 States and UT of A&N Islands. Which of the statements given above is/are correct? (a) 1 only (b) 2 only (c) Both 1 and 2 (d) Neither 1 nor 2

Solution: B

75 tribal groups have been categorized by Ministry of Home Affairs as Particularly Vulnerable Tribal Groups (PVTG)s. PVTGs reside in 18 States and UT of A&N Islands. The Ministry of Tribal Affairs implements the Scheme of “Development of Particularly Vulnerable Tribal Groups (PVTGs)” exclusively for them. Under the scheme, Conservation-cum-Development (CCD)/Annual Plans are to be prepared by each State/UT for their PVTGs based on their need assessment, which are then appraised and approved by the Project Appraisal Committee of the Ministry. Activities for development of PVTGs are undertaken in Sectors of Education, Health, Livelihood www.insightsonindia.com 58 INSTA Revision 3.0 INSIGHTSIAS SIMPLYFYING IAS EXAM PREPARATION and Skill Development, Agricultural Development , Housing & Habitat, Conservation of Culture etc. https://tribal.nic.in/pvtg.aspx

77. Which of the following criteria is followed for determination of PVTGs? 1. A pre-agriculture level of technology. 2. A stagnant or declining population. 3. Extremely low literacy. 4. A subsistence level of economy. Select the correct answer using the code given below: (a) 1, 2 and 3 only (b) 1, 2 and 4 only (c) 2, 3 and 4 only (d) 1, 2, 3 and 4

Solution: D

About ‘Particularly Vulnerable Tribal Groups (PVTGs)’:

• PVTGs are more vulnerable among the tribal groups.

• They have declining or stagnant population, low level of literacy, pre-agricultural level of technology and are economically backward.

• They generally inhabit remote localities having poor infrastructure and administrative support. Identification:

• In 1975, the Government of India initiated to identify the most vulnerable tribal groups as a separate category called PVTGs and declared 52 such groups, while in 1993 an additional 23 groups were added to the category, making it a total of 75 PVTGs out of 705 Scheduled Tribes, spread over 18 states and one Union Territory (A&N Islands) in the country (2011 census).

• Among the 75 listed PVTG’s the highest number are found in Odisha (13), followed by Andhra Pradesh (12). Scheme for development of PVTGs:

• The Ministry of Tribal Affairs implements the Scheme of “Development of Particularly Vulnerable Tribal Groups (PVTGs)” exclusively for them.

• Under the scheme, Conservation-cum-Development (CCD)/Annual Plans are to be prepared by each State/UT for their PVTGs based on their need assessment, www.insightsonindia.com 59 INSTA Revision 3.0 INSIGHTSIAS SIMPLYFYING IAS EXAM PREPARATION

which are then appraised and approved by the Project Appraisal Committee of the Ministry.

• Priority is also assigned to PVTGs under the schemes of Special Central Assistance (SCA) to Tribal Sub-Scheme(TSS), Grants under Article 275(1) of the Constitution, Grants-in-aid to Voluntary Organisations working for the welfare of Schedule Tribes and Strengthening of Education among ST Girls in Low Literacy Districts. The criteria followed for determination of PVTGs are as under: 1. A pre-agriculture level of technology. 2. A stagnant or declining population. 3. Extremely low literacy. 4. A subsistence level of economy.

78. Consider the following statements regarding the administration of union territory Puducherry: 1. Legislative Assembly can disapprove the ordinance passed by the administrator (LG) in Puducherry. 2. Puducherry’s elected government cannot make laws concerning public order, police and land rights. Which of the statements given above is/are correct? (a) 1 only (b) Gulf of Oman (c) Both 1 and 2 (d) Neither 1 nor 2

Solution: A

• Puducherry is a union territory which is governed by Article 239A of the Constitution.

• NCT of Delhi, meanwhile, is governed by Article 239AA of the Constitution which imposes restrictions on the law-making power of the elected legislature of Delhi.

• According to Article 239AA, the elected legislature of NCT of Delhi cannot enact laws relating to entries 1,2 and 18 of the state list.

• It means that Delhi’s elected government cannot make laws concerning public order, police and land rights. This is not the case with Puducherry.

www.insightsonindia.com 60 INSTA Revision 3.0 INSIGHTSIAS SIMPLYFYING IAS EXAM PREPARATION

• According to 239A which governs the administration of Puducherry, the administrator (LG) does not have power to promulgate any ordinance in Puducherry.

• Even if LG does it, it will not stand if the Legislative Assembly decides to disapprove it. This emphasizes the supremacy of the legislature over the administrator in Puducherry, unlike the restrictions on the government of NCT Delhi.

• According to Article 240, even the rights of the President of India is curtailed to the point that he/she cannot issue ordinances to suit their whims and fancies.

79. Consider the following pairs of PVTGs and the states they reside 1. Siddi : Gujarat 2. Jenu Kuruba : Karnataka 3. Kadar : Andhra Pradesh Which of the pairs given above is/are correctly matched? (a) 1 and 2 only (b) 1 and 3 only (c) 2 and 3 only (d) 1, 2 and 3

Solution: A

The Siddi are recognized as a scheduled tribe in 3 states and 1 union territory: Goa, Gujarat, Karnataka and Daman and Diu. The Jennu Kurumba are spread out mainly along the border forests of Kerala, Tamil Nadu and Karnataka. They are also referred to by the names ‘Then Kurumba’ or ‘Kattu Naikar.’

www.insightsonindia.com 61 INSTA Revision 3.0 INSIGHTSIAS SIMPLYFYING IAS EXAM PREPARATION

80. Chitwan National Park, sometimes seen in news, located in: (a) Bhutan (b) Bangladesh (c) Nepal (d) Myanmar

Solution: C

Chitwan National Park is a preserved area in the Terai Lowlands of south-central Nepal, known for its biodiversity. Its dense forests and grassy plains are home to rare mammals like one-horned rhinos and Bengal tigers. The park shelters numerous bird species, including the giant hornbill. Dugout canoes traverse the northern Rapti River, home to crocodiles. Inside the park is Balmiki Ashram, a Hindu site. https://www.thehindu.com/news/national/no-indo-nepal-pact-on-tigers- yet/article26122318.ece

81. Which of the following countries border Baltic Sea? 1. Norway 2. Sweden 3. Latvia 4. Poland Select the correct answer using the code given below: (a) 1, 2 and 3 only (b) 1, 2 and 4 only (c) 2, 3 and 4 only (d) 1, 2, 3 and 4

Solution: C

www.insightsonindia.com 62 INSTA Revision 3.0 INSIGHTSIAS SIMPLYFYING IAS EXAM PREPARATION

82. Consider the following statements regarding the Brus tribes: 1. The Brus, are spread across the northeastern states of Tripura, Assam, Manipur, and Mizoram. 2. In Tripura, they are recognized as a Particularly Vulnerable Tribal Group. 3. In Mizoram, they have been targeted by groups that do not consider them indigenous to the state. Which of the statements given above is/are correct? (a) 1 and 2 only (b) 1 and 3 only www.insightsonindia.com 63 INSTA Revision 3.0 INSIGHTSIAS SIMPLYFYING IAS EXAM PREPARATION

(c) 2 and 3 only (d) 1, 2 and 3

Solution: D

The centre has signed a historic pact for permanent solution of Bru refugees’ issue. The agreement is between Union Government, Governments of Tripura and Mizoram and Bru-Reang representatives to end the 23-year old Bru-Reang refugee crisis. Highlights of the agreement:

• Under the agreement, the centre has announced a package of Rs. 600 crores under this agreement.

• As per the agreement the Bru tribes would be given land to reside in Tripura.

• A fixed deposit of Rs. 4 lakh will be given to each family as an amount of government aid. They will be able to withdraw this amount after two years.

• Each of the displaced families will be given 40×30 sq ft residential plots.

• Apart from them, each family will be given Rs. 5,000 cash per month for two years.

• The agreement highlights that each displaced family will also be given free ration for two years and aid of Rs. 1.5 lakh to build their houses. Significance of the government:

• This agreement will bring a permanent solution for the rehabilitation of thousands of Bru-Reang people in Tripura. The government believes that this agreement will bring a bright future for them. Bru-Reang people will be able to enjoy the benefits of all social-welfare schemes of governments. Background:

• More than 30,000 Bru tribes who fled Mizoram, are residing in Tripura’s refugee camps. Who are Brus?

• The Brus, also referred to as the Reangs, are spread across the northeastern states of Tripura, Assam, Manipur, and Mizoram.

• In Tripura, they are recognised as a Particularly Vulnerable Tribal Group. In Mizoram, they have been targeted by groups that do not consider them indigenous to the state.

www.insightsonindia.com 64 INSTA Revision 3.0 INSIGHTSIAS SIMPLYFYING IAS EXAM PREPARATION

What’s the issue?

• A bout of ethnic violence forced thousands of people from the Bru tribe to leave their homes in Mizoram.

• The displaced Bru people from Mizoram have been living in various camps in Tripura since 1997. In 1997, the murder of a Mizo forest guard at the Dampa Tiger Reserve in Mizoram’s Mamit district allegedly by Bru militants led to a violent backlash against the community, forcing several thousand people to flee to neighbouring Tripura.

• The Bru militancy was a reactionary movement against Mizo nationalist groups who had demanded in the mid-1990s that the Brus be left out of the state’s electoral rolls, contending that the tribe was not indigenous to Mizoram.

83. Which of the following countries is/are members of the Arctic Council: 1. France 2. Canada 3. Russia 4. Germany Select the correct answer using the code given below: (a) 1 and 4 only (b) 2 and 3 only (c) 1, 2 and 3 only (d) 2, 3 and 4 only

Solution: B

The Arctic Council is the leading intergovernmental forum promoting cooperation, coordination and interaction among the Arctic States, Arctic indigenous communities and other Arctic inhabitants on common Arctic issues, in particular on issues of sustainable development and environmental protection in the Arctic.

• It was formally established in 1996.

• The Ottawa Declaration defines these states as Members of the Arctic Council: Canada, Denmark, Finland, Iceland, Norway, Russia, Sweden and the United States.

• India is an observer member of the council. https://indianexpress.com/article/explained/india-re-election-observer-arctic- council-importance-5727126/

www.insightsonindia.com 65 INSTA Revision 3.0 INSIGHTSIAS SIMPLYFYING IAS EXAM PREPARATION

84. Consider the following statements regarding Eklavya Model Residential Schools 1. Eklavya Model Residential Schools (EMRS) are set up in States/UTs with grants under Article 275(1) of the Constitution of India. 2. The scheme is being implemented by the Ministry of Tribal Affairs. 3. As per the budget 2018-19, every block with more than 75% ST population and at least 20,000 tribal persons will have an Eklavya Model Residential School by 2022. Which of the statements given above is/are correct? (a) 1 and 2 only (b) 1 and 3 only (c) 2 and 3 only (d) 1, 2 and 3

Solution: A

Eklavya Model Residential Schools

• In the context of establishing quality residential schools for the promotion of education, Eklavya Model Residential Schools (EMRSs) for ST students are set up in States/UTs with provisioning of funds through “Grants under Article 275(1) of the Constitution”.

• The establishing of EMRSs is based on demand of the concerned States/UTs with availability of land as an essential attribute.

• As per the budget 2018-19, every block with more than 50% ST population and at least 20,000 tribal persons will have an Eklavya Model Residential School by 2022. Background: What is Eklavya Model Residential School (EMRS)?

• EMRS is a Government of India scheme for model residential school for Indian tribals (Scheduled Tribes, ST) across India.

• In the context of the trend of establishing quality residential schools for the promotion of education in all areas and habitations in the country, the Eklavya Model Residential Schools (EMRS) for ST students take their place among the Jawahar Navodaya Vidyalayas, the Kasturba Gandhi Balika Vidyalayas and the Kendriya Vidyalayas. Eklavya Model Residential Schools (EMRS) are set up in States/UTs with grants under Article 275(1) of the Constitution of India.

• The scheme is being implemented by the Ministry of Tribal Affairs, Government of India.

www.insightsonindia.com 66 INSTA Revision 3.0 INSIGHTSIAS SIMPLYFYING IAS EXAM PREPARATION

Objectives of EMRS:

• Comprehensive physical, mental and socially relevant development of all students enrolled in each and every EMRS. Students will be empowered to be change agent, beginning in their school, in their homes, in their village and finally in a larger context.

• Focus differentially on the educational support to be made available to those in Standards XI and XII, and those in standards VI to X, so that their distinctive needs can be met,

• Support the annual running expenses in a manner that offers reasonable remuneration to the staff and upkeep of the facilities.

• Support the construction of infrastructure that provides education, physical, environmental and cultural needs of student life.

85. Consider the following statements regarding the 12th century social reformer Basavanna: 1. He is associated with the emergence of Vaishnavism in Karnataka. 2. He was against the idol worship. Which of the statements given above is/are correct? (a) 1 only (b) 2 only (c) Both 1 and 2 (d) Neither 1 nor 2

Solution: B

The Virashaiva movement was initiated by Basavanna and his companions like Allama Prabhu and Akkamahadevi. This movement began in Karnataka in the mid- twelfth century. The Virashaivas argued strongly for the equality of all human beings and against Brahmanical ideas about caste and the treatment of women. They were also against all forms of ritual and idol worship.

86. Consider the following statements regarding Sentinelese 1. The Sentinelese are a negrito tribe who live on the North Sentinel Island of the Andamans. 2. Their numbers are believed to be less than 150 and as low as 40. 3. Genome studies indicate that the Andaman tribes could have been on the islands even 30,000 years ago. www.insightsonindia.com 67 INSTA Revision 3.0 INSIGHTSIAS SIMPLYFYING IAS EXAM PREPARATION

Which of the statements given above is/are correct? (a) 1 and 2 only (b) 1 and 3 only (c) 2 and 3 only (d) 1, 2 and 3

Solution: D

• The Government has promulgated various laws/regulations from time to time to ensure that the rights and well-being of the Sentinelese are safeguarded. Steps taken to ensure the protection of Sentinelese:

• The entire North Sentinel Island along with 5 km coastal sea from high water mark is notified as tribal reserve.

• The Government respects their way of life style, therefore, has adopted an ‘eyes- on and hands-off’ practice to protect and safeguard the Sentinelese tribe.

• A protocol of circumnavigation of the North Sentinel Island has been notified. The ships and aircrafts of Coast Guard and boats of Marine Police make sorties around North Sentinel to keep surveillance. They have been protected under:

• A &N Islands (PAT) Regulation 1956.

• Scheduled Castes and the Scheduled Tribes (Prevention of Atrocities) Act, 1989.

• Restrictions under Foreigner (Restricted Area) Orders, 1963.

• Visa Manual Conditions/Passport Act 1920, Indian Forest Act, 1927 and Wildlife (Protection) Act, 1972. Who are the Sentinelese?

• The Sentinelese are a negrito tribe who live on the North Sentinel Island of the Andamans. The inhabitants are connected to the Jarawa on the basis of physical, as well as linguistic similarities. Their numbers are believed to be less than 150 and as low as 40.

• Based on carbon dating of kitchen middens by the Anthropological Survey of India, Sentinelese presence was confirmed in the islands to 2,000 years ago. Genome studies indicate that the Andaman tribes could have been on the islands even 30,000 years ago. Why are they said to be vulnerable?

• It is said they have made little to no advancement in the over 60,000 years and still live very primitive lives, surviving mainly on fish and coconuts. www.insightsonindia.com 68 INSTA Revision 3.0 INSIGHTSIAS SIMPLYFYING IAS EXAM PREPARATION

• They are very vulnerable to germs since they have not had contact with the outside world. Even a common flu virus carried by a visitor could wipe out the entire tribe.

• Since the 1960s, there have been a handful of efforts to reach out to the tribe but all have largely failed. They have repeatedly, aggressively made it clear that they want to be isolated. Conclusion:

• Currently, there is a one-size-fits-all policy. For instance, the Sentinelese should be left alone. The rights and the desires of the Sentinelese need to be respected and nothing is to be achieved by escalating the conflict and tension.

87. Consider the following statements regarding tribes on Lakshadweep islands 1. Lakshadweep is Muslim majority state in India with approximately 96.58 % of state population following Islam as their religion. 2. Of the total population of Lakshadweep, 94.5 per cent are Scheduled Tribes (STs). Which of the statements given above is/are correct? (a) 1 only (b) 2 only (c) Both 1 and 2 (d) Neither 1 nor 2

Solution: C

Lakshadweep is Muslim majority state in India with approximately 96.58 % of state population following Islam as their religion. is second most popular religion in state of Lakshadweep with approximately 2.77 % following it. In Lakshadweep state, Christianity is followed by 0.49 %, Jainism by 0.02 %, by 0.02 % and Sikhism by 0.01 %. Around 0.01 % stated ‘Other Religion’, approximately 0.10 % stated ‘No Particular Religion’. Literacy rate in Lakshadweep has seen upward trend and is 91.85 percent as per 2011 population census. Of that, male literacy stands at 95.56 percent while female literacy is at 87.95 percent. In 2001, literacy rate in Lakshadweep stood at 86.66 percent of which male and female were 92.53 percent and 80.47 percent literate respectively. Total area of Lakshadweep is 30 sq. km. Density of Lakshadweep is 2,149 per sq km which is higher than national average 382 per sq km. In 2001, density of Lakshadweep was 1,895 per sq km, while nation average in 2001 was 324 per sq km.

www.insightsonindia.com 69 INSTA Revision 3.0 INSIGHTSIAS SIMPLYFYING IAS EXAM PREPARATION

The total population of Lakshadweep as per the 2001 Census is 60,650. Of this, 57,321 persons (94.5 per cent) are Scheduled Tribes (STs). Among the states/UTs Lakshadweep has the highest proportion of ST population besides Mizoram. According to the 2011 Census, Lakshadweep has a population of 64429 persons. More than 93% of the population who are indigenous, are Muslims and majority of them belong to the Shafi School of the Sunni Sect. Malayalam is spoken in all the islands except Minicoy where people speak Mahl which is written in Divehi script and is spoken in Maldives also. The entire indigenous population has been classified as Scheduled Tribes because of their economic and social backwardness. According to the Scheduled Castes and Scheduled Tribes list (modification orders), 1956, the inhabitants of Lakshadweep who and both of whose parents were born in these islands are treated as Scheduled Tribes. There are no Scheduled Castes in this Union Territory.

88. Consider the following statements regarding the Trans Fats: 1. They can be produced by heating liquid vegetable oils in the presence of hydrogen gas and a catalyst. 2. It raises the bad cholesterol level in the body. 3. It can contribute to insulin resistance. Which of the statements given above is/are correct? (a) 1 and 2 only (b) 2 and 3 only (c) 1 and 3 only (d) 1, 2 and 3

Solution: D

Trans fatty acids, more commonly called trans fats, are made by heating liquid vegetable oils in the presence of hydrogen gas and a catalyst, a process called hydrogenation. Partially hydrogenating vegetable oils makes them more stable and less likely to become rancid. This process also converts the oil into a solid, which makes them function as margarine or shortening. Partially hydrogenated oils can withstand repeated heating without breaking down, making them ideal for frying fast foods. For these reasons, partially hydrogenated oils became a mainstay in restaurants and the food industry – for frying, baked goods, and processed snack foods and margarine. Partially hydrogenated oil is not the only source of trans fats in our diets. Trans fats are also naturally found in beef fat and dairy fat in small amounts. www.insightsonindia.com 70 INSTA Revision 3.0 INSIGHTSIAS SIMPLYFYING IAS EXAM PREPARATION

Trans fats are the worst type of fat for the heart, blood vessels, and rest of the body because they:

• Raise bad LDL (Bad cholesterol) and lower good HDL

• Create inflammation,– a reaction related to immunity – which has been implicated in heart disease, stroke, diabetes, and other chronic conditions

• Contribute to insulin resistance

• Can have harmful health effects even in small amounts – for each additional 2 percent of calories from trans fat consumed daily, the risk of coronary heart disease increases by 23 percent. https://www.thehindu.com/sci-tech/health/who-for-eliminating-industrially- produced-trans-fats-by-2023/article27077068.ece

89. Which of the following countries border Mediterranean Sea 1. Italy 2. Portugal 3. Algeria 4. Tunisia Select the correct answer using the code given below: (a) 1 and 2 only (b) 1, 3 and 4 only (c) 2, 3 and 4 only (d) 1, 2, 3 and 4

Solution: B

www.insightsonindia.com 71 INSTA Revision 3.0 INSIGHTSIAS SIMPLYFYING IAS EXAM PREPARATION

90. Consider the following statements regarding the Bureau of Indian Standards: 1. It is the National Standard Body of India established under the BIS Act 2016. 2. It works under the aegis of the Ministry of Consumer Affairs, Food & Public Distribution, Government of India. Which of the statements given above is/are correct? (a) 1 only (b) 2 only (c) Both 1 and 2 (d) Neither 1 nor 2

Solution: C

BIS is the National Standard Body of India established under the BIS Act 2016 for the harmonious development of the activities of standardization, marking and quality certification of goods and for matters connected therewith or incidental thereto. BIS has been providing traceability and tangibility benefits to the national economy in a number of ways – providing safe reliable quality goods; minimizing health hazards to consumers; promoting exports and imports substitute; control over proliferation of varieties etc. through standardization, certification and testing. It works under the aegis of the Ministry of Consumer Affairs, Food & Public Distribution, Government of India.

www.insightsonindia.com 72 INSTA Revision 3.0 INSIGHTSIAS SIMPLYFYING IAS EXAM PREPARATION

Keeping in view, the interest of consumers as well as the industry, BIS is involved in various activities as given below:

• Standards Formulation • Laboratory Recognition Scheme • Product Certification Scheme • Sale of Indian Standards • Compulsory Registration Scheme • Consumer Affairs Activities • Foreign Manufacturers Certification • Promotional Activities Scheme • Training Services, National & • Hall Marking Scheme International level • Laboratory Services • Information Services

91. Consider the following physical features of Africa 1. Atlas Mountains 2. Ethiopian Highlands 3. Congo Basin Arrange the above given features from West to East using the code given below: (a) 1-3-2 (b) 3-2-1 (c) 2-3-1 (d) 2-1-3

Solution: A

www.insightsonindia.com 73 INSTA Revision 3.0 INSIGHTSIAS SIMPLYFYING IAS EXAM PREPARATION

92. Consider the following statements regarding Tangam tribal community: 1. They reside in the state of Nagaland. 2. Their language Tangam belongs to the Dravidian language family. www.insightsonindia.com 74 INSTA Revision 3.0 INSIGHTSIAS SIMPLYFYING IAS EXAM PREPARATION

Which of the statements given above is/are correct? (a) 1 only (b) 2 only (c) Both 1 and 2 (d) Neither 1 nor 2

Solution: D

The Tangams are a little-known community within the larger Adi tribe of Arunachal Pradesh and reside in the hamlet of Kugging in Upper Siang district’s Paindem circle. As per the UNESCO World Atlas of Endangered Languages (2009), Tangam — an oral language that belongs to the Tani group, under the greater Tibeto-Burman language family — is marked ‘critically endangered’. https://indianexpress.com/article/explained/the-language-of-the-tangams-with- just-253-speakers-6503165/

93. Consider the following statements regarding the Basel Convention: 1. It is an independent intergovernmental body. 2. The WCO Secretariat runs the WCO’s day-to-day operations under the leadership of a Secretary General. Which of the statements given above is/are correct? (a) 1 only (b) 2 only (c) Both 1 and 2 (d) Neither 1 nor 2

Solution: C

The World Customs Organization (WCO), established in 1952 as the Customs Co- operation Council (CCC) is an independent intergovernmental body whose mission is to enhance the effectiveness and efficiency of Customs administrations. Today, the WCO represents 183 Customs administrations across the globe that collectively process approximately 98% of world trade. As the global centre of Customs expertise, the WCO is the only international organization with competence in Customs matters and can rightly call itself the voice of the international Customs community. The WCO’s governing body is the Council. www.insightsonindia.com 75 INSTA Revision 3.0 INSIGHTSIAS SIMPLYFYING IAS EXAM PREPARATION

The WCO Secretariat, which is based in Brussels, Belgium, consists of more than 125 staffers from around the world and runs the WCO’s day-to-day operations under the leadership of a Secretary General.

94. Rabari, Bharvad and Charan tribal communities, recently seen in news, reside in the state of (a) Karnataka (b) Gujarat (c) Andhra Pradesh (d) Kerala

Solution: B

The Gujarat government announced that a five-member commission will be formed to identify the members of Rabari, Bharvad and Charan communities, living in nesses (tiny, oval-shaped hutments made of mud) of Gir, Barda and Alech areas of the state, who are eligible to get the benefits of Schedule Tribe (ST) status. The commission, which will be headed by a retired judge of the high court, comprises two district judges, one retired forest officer and one retired revenue officer. https://indianexpress.com/article/india/gujarat-panel-to-be-formed-to-decide-on- tribal-status-of-3-communities-6497045/

95. Consider the following statements regarding the monkeypox disease: 1. It is a viral zoonotic disease that occurs primarily in tropical rainforest areas of Central and West Africa. 2. Human-to-human transmission of the virus do not occur. 3. There is currently no specific treatment recommended for monkeypox Which of the statements given above is/are correct? (a) 1 only (b) 2 and 3 only (c) 1 and 3 only (d) 2 only

Solution: A

www.insightsonindia.com 76 INSTA Revision 3.0 INSIGHTSIAS SIMPLYFYING IAS EXAM PREPARATION

Monkeypox is caused by monkeypox virus, a member of the Orthopoxvirus genus in the family Poxviridae.

• Monkeypox is a viral zoonotic disease that occurs primarily in tropical rainforest areas of Central and West Africa and is occasionally exported to other regions.

• Monkeypox typically presents clinically with fever, rash and swollen lymph nodes.

• Monkeypox virus is mostly transmitted to people from wild animals such as rodents and primates, but human-to-human transmission also occurs.

• Monkeypox virus is transmitted from one person to another by contact with lesions, body fluids, respiratory droplets and contaminated materials such as bedding.

• Typically, up to a tenth of persons ill with monkeypox may die, with most deaths occurring in younger age groups.

• The clinical presentation of monkeypox resembles that of smallpox, a related orthopoxvirus infection which was declared eradicated worldwide in 1980.

• There is currently no specific treatment recommended for monkeypox. Vaccination against smallpox with vaccinia vaccine was demonstrated through several observational studies to be about 85% effective in preventing monkeypox. Thus, prior childhood smallpox vaccination may result in a milder disease course.

• However at the present time, the original (first-generation) smallpox vaccines are no longer available to the general public. A newer vaccinia-based vaccine was approved for the prevention of smallpox and monkeypox in 2019 and is also not yet widely available in the public sector.

96. Consider the following statements regarding Katkari Tribes: 1. It is one of the Particularly Vulnerable Tribal Groups. 2. It is a primitive tribe found in Maharashtra and parts of Gujarat, Which of the statements given above is/are correct? (a) 1 only (b) 2 only (c) Both 1 and 2 (d) Neither 1 nor 2

Solution: C

www.insightsonindia.com 77 INSTA Revision 3.0 INSIGHTSIAS SIMPLYFYING IAS EXAM PREPARATION

Katkari is one of the 75 Particularly Vulnerable Tribal Groups, as per the classification by Ministry of Home Affairs. It is a primitive tribe found in Maharashtra (Pune, Raigad and Thane districts) and parts of Gujarat. Katkaris are also known as Kathodis because of their old occupation of making Katha (Catechu) the thickened sap from wood of Khair ( Acacia catechu). They serve as agricultural labourers and sell firewood and some jungle fruits. They also take up fishing for domestic consumption, coal making and brick manufacturing. Poor literacy rates, health conditions and minimal livelihood opportunities are some of the issues faced by the tribal people. https://vikaspedia.in/social-welfare/scheduled-tribes-welfare/the-story-of- shahapurs-katkari-tribe

97. Nuakhai is an agricultural festival mainly observed in the state of: (a) Karnataka (b) Odisha (c) Andhra Pradesh (d) Jharkhand

Solution: B

Nuakhai or Navakhai is an agricultural festival mainly observed by people of Western Odisha and Southern Chhattisgarh in India. Nuakhai is observed to welcome the new rice of the season. https://www.hindustantimes.com/india-news/pm-modi-extends-greeting-to- farmers-on-nuakhai-juhar/story-4l9IcrPrkOJkhpfCMxEaML.html

98. Consider the following statements regarding Asiatic Lion species: 1. Its current range is restricted to the Gir National Park and environs in the Indian state of Gujarat. 2. It is listed in Schedule I of Wildlife (Protection) Act 1972. 3. It is listed as endangered on IUCN Red List. Which of the statements given above is/are correct? (a) 1 and 2 only (b) 2 and 3 only (c) 1 and 3 only (d) 1, 2 and 3

www.insightsonindia.com 78 INSTA Revision 3.0 INSIGHTSIAS SIMPLYFYING IAS EXAM PREPARATION

Solution: D

Asiatic lions were once distributed upto the state of West Bengal in east and Rewa in Madhya Pradesh, in central India. At present Gir National Park and Wildlife Sanctuary is the only abode of the Asiatic lion. The last surviving population of the Asiatic lions is a compact tract of dry deciduous forest and open grassy scrublands in southwestern part of Saurashtra region of Gujarat. Conservation Status:

• Listed in Schedule I of Wildlife (Protection) Act 1972

• Appendix I of CITES

• Endangered on IUCN Red List https://www.thehindu.com/sci-tech/science/ccmb-scientists-sequence-asiatic- lion-genome/article27103990.ece https://www.wwfindia.org/about_wwf/priority_species/threatened_species/asiatic _lion/

99. Consider the following pairs of festivals and tribes associated with: Festival/Fairs Tribe 1. Thisam festival : Nagas 2. Baneshwar Fair : Bhils 3. Myoko Festival : Apatanis Which of the pairs given above is/are correctly matched? (a) 1 and 2 only (b) 2 and 3 only (c) 1 and 3 only (d) 1, 2 and 3

Solution: D

Thisam Phanit is a cultural practice of celebrating the festival of the dead. It is one of the major festivals of the Tangkhuls and the Naga Tribes of Manipur, India. It is observed as an emotional farewell festival to the spirit of the dead, for 12 days at the end of January every year. Baneshwar fair is an annual tribal fair held in Dungarpur district in Rajasthan state of India. The fair is held in the month of January or February at Baneshwar, near the confluence of the Som and Mahi rivers. This fair is a major fair in tribal culture and has been described as “the Kumbh mela for the tribals”. It is predominantly a tribal fair with more than half of the congregation consisting of Bhils. They revere Baneshwar Mahadev as well as Mavji. www.insightsonindia.com 79 INSTA Revision 3.0 INSIGHTSIAS SIMPLYFYING IAS EXAM PREPARATION

Myoko is a celebration of friendship and harmony between various Apatani villages. This is also the time when new paddy is sown in the terraced fields and the Apatani pray for a good crop. The festival rotates between the eight Apatani villages in the state of Arunachal Pradesh.

100. Consider the following statements regarding the Index of Eight Core Industries: 1. It is compiled and released by the Central Statistics Office (CSO) 2. The base year for the Index currently is 2004-05. Which of the statements given above is/are correct? (a) 1 only (b) 2 only (c) Both 1 and 2 (d) Neither 1 nor 2

Solution: D

The monthly Index of Eight Core Industries (ICI) is a production volume index. ICI measures collective and individual performance of production in selected eight core industries viz. Coal, Crude Oil, Natural Gas, Refinery Products, Fertilizers, Steel, Cement and Electricity. It is compiled and released by Office of the Economic Adviser (OEA), Department of Industrial Policy & Promotion (DIPP), and Ministry of Commerce & Industry. The inter-se weights of these eight industries are largely in alignment with the respective weight of these industries in the Index of Industrial Production (IIP). The base year of the ICI has been revised to 2011-12 from 2004-05 in alignment with the new series of IIP. Combined weight of these eight core industries is 40.27 percent of IIP with base 2011-12.

www.insightsonindia.com 80 INSTA Revision 3.0